Orthognathic, TMJ, Chin Flashcards

1
Q

Which of the following is the most common etiology of ankylosis of the temporomandibular joint?

(A) Autoimmune
(B) Congenital
(C) Infectious
(D) Neoplastic
(E) Trauma

A

The correct response is Option E.

Intra-articular (true) ankylosis of the temporomandibular joint (TMJ) most frequently occurs as a result of trauma. In patients with true ankylosis, destruction of the articular disk and joint elements occurs, leading to fibrosis, narrowing of the joint space, and ultimately bony fusion.

Abnormalities of bone, cartilage, and/or soft tissue have also been shown to cause TMJ ankylosis.

Other causes of TMJ ankylosis, such as congenital abnormalities, idiopathic occurrences, infection, and juvenile rheumatoid arthritis, have been documented but are less common than trauma. Neoplasms of the TMJ are extremely rare.

How well did you know this?
1
Not at all
2
3
4
5
Perfectly
2
Q

In a 20-year old man, examination of the occlusion shows that the mesial buccal cusp of the maxillary first molar articulates between the first and second mandibular molars. Cephalometric analysis shows SNA angle of 70 degrees (normal 80 to 82 degrees) and SNB angle of 79 degrees (normal 79 to 80 degrees). Which of the following is the most likely diagnosis?

(A) Long face syndrome

(B) Maxillary deficiency

(C) Maxillary excess

(D) Retrognathia

(E) True prognathism

A

The correct response is Option B.

In a Class III mesioclusion, the mesial buccal cusp of the maxillary first molar articulates between the first and second mandibular molars. The mandible is mesially placed or prognathic compared to the maxilla. This type of malocclusion may result from maxillary deficiency, mandibular overgrowth, or a combination of both.

Long face syndrome results from vertical maxillary excess. Physical findings include excessive gingival and upper incisor show at rest and during smiling, a long vertical face, and a retrognathic mandible caused by backward autorotation or true retrognathism.

At rest, normal incisor show is approximately 2 to 3 mm. Decreased incisor show usually indicates maxillary hypoplasia. Excessive gingival show indicates excess maxillary growth.

This patient has a normal SNB angle, indicating that he does not have true prognathism but that it only appears excessive because the maxilla is retrusive. The SNA angle (normally 82 degrees) relates the maxilla to the cranial base. The SNB angle (normally 79 degrees) relates the mandible to the cranial base.

How well did you know this?
1
Not at all
2
3
4
5
Perfectly
3
Q

A 25-year-old healthy man presents with a painful clicking when opening and closing his mouth 12 weeks after being involved in a physical altercation. He is able to chew and open and close his mouth normally, but with discomfort. Anteroposterior x-ray study shows no abnormalities. Which of the following is the most likely source of his discomfort?

A) Articular disc subluxation
B) Dynamic condylar subluxation
C) Early arthritis
D) Occult fracture of the condylar head
E) Spasm of the lateral pterygoid muscle

A

The correct response is Option A.

This patient likely has increased mobility of the articular disc. This can occur as a result of acute trauma (as in this case) or chronic trauma, such as bruxism. At this juncture, the disc is reducing with motion, so there is no obstruction to movement. Nevertheless, symptoms can worsen over time and create a closed-lock wherein the patient cannot open his mouth. An MRI and/or ultrasound can help confirm the pathology. With the limited and nonmechanical symptoms (eg, locking), treatment is conservative.

Arthritis is possible but unlikely in a patient of this age, especially without some other reason, such as infection or a history of juvenile rheumatoid arthritis. Fracture of the condylar head is possible, but this should have healed after 2 months and would be asymptomatic. Subluxation of the condylar head would restrict motion, and spasm of the lateral pterygoid can cause temporomandibular joint pain, but subluxation of the condylar head does not produce the click that is heard.

How well did you know this?
1
Not at all
2
3
4
5
Perfectly
4
Q

Which of the following is the optimal amount of incisor show at rest?

(A) 0 to 1 mm
(B) 2 to 3 mm
(C) 4 to 5 mm
(D) 6 to 7 mm

A

The correct response is Option B.

At rest, the upper lips should be parted slightly and 2 to 3 mm of upper central incisors should be visible beneath the lower border of the upper lip. Only 1 to 2 mm of gingiva should be visible during a full smile. Patients with vertical maxillary deficiency typically have no incisor show at rest, resulting in a “prematurely aged” appearance. Patients with vertical maxillary excess often exhibit a “gummy” smile that occurs as a result of excessive gingival show.

How well did you know this?
1
Not at all
2
3
4
5
Perfectly
5
Q

Which of the following sites is osteotomized in the Le Fort III osteotomy but NOT in the monobloc advancement osteotomy?

(A) Frontozygomatic suture
(B) Inferior orbital fissure
(C) Lamina papyracea
(D) Pterygomaxillary fissure
(E) Zygomatic arch

A

The correct response is Option A.

Although the Le Fort III osteotomy, as shown in the first illustration on page 36, is most often used for correction of midface hypoplasia in patients with craniosynostosis, the monobloc advancement osteotomy is now gaining acceptance. In the Le Fort III procedure, the osteotomies pass through the nasofrontal junction first, then laterally across the medial orbital wall (lamina papyracea) and onto the orbital floor, continuing into the inferior orbital fissure. The lateral orbital wall is cut through the frontozygomatic suture and separated from the cranium; it continues inferiorly and posteriorly, where a pterygomaxillary disjunction is performed. After the zygomatic arch is cut, the advancement can be performed.

In the monobloc advancement, shown in the second illustration on page 36, the osteotomy lines are similar to the Le Fort III osteotomy, but the nasofrontal junction and frontozygomatic suture are not osteotomized. An advantage of the monobloc procedure is simultaneous correction of the supraorbital and midface deformities. However, this technique is associated with higher rates of infection and cerebrospinal fluid leakage, which is most likely due to direct communication between the cranial and nasal cavities.

How well did you know this?
1
Not at all
2
3
4
5
Perfectly
6
Q

In patients with Treacher Collins syndrome, which of the following is a characteristic skeletal finding?

(A) Brachycephaly
(B) Hypertelorism
(C) Macrogenia
(D) Malar hypoplasia
(E) Preaxial polysyndactyly

A
The correct response is Option D.
The characteristic skeletal finding in patients with Treacher Collins syndrome is hypoplasia of the malar bones, which often occurs in conjunction with clefting through the zygomatic arches. Patients also have hypoplasia of the maxilla and mandible and antegonial notching of the angle of the mandible. Occlusion is Angle class II; there is an anterior open bite and clockwise rotation of the occlusal plane. Effects on the temporomandibular joint are varied.

Brachycephaly, macrogenia, preaxial polysyndactyly, and hypertelorism do not occur in patients with Treacher Collins syndrome.

How well did you know this?
1
Not at all
2
3
4
5
Perfectly
7
Q

In patients with vertical maxillary excess undergoing Le Fort osteotomy with maxillary impaction, which of the following findings is most likely postoperatively?
(A) Increased mentalis strain
(B) Increased upper incisal show
(C) More obtuse nasolabial angle
(D) Retrogenia
(E) Widened alar base

A

The correct response is Option E.

Patients with vertical maxillary excess, or long face syndrome, have a narrow alar base, an obtuse nasolabial angle, and an anterior open bite. Mentalis muscle strain and labial incompetence are increased, and there is excess gingival show and exposure of the upper incisors.

Appropriate management is Le Fort I osteotomy with maxillary impaction; osseous genioplasty is also performed in some patients. These procedures will correct many of the findings associated with this condition, including decreasing the mentalis muscle strain and incisal show and creating a more acute nasolabial angle. The alar base will be widened. Le Fort I osteotomy also rotates the mandible forward and upward, resolving the retrogenia associated with long face syndrome. Postoperative lateral cephalograms will show forward autorotation of the mandible with counterclockwise rotation.

How well did you know this?
1
Not at all
2
3
4
5
Perfectly
8
Q

The lateral cephalogram shown above is from a 16-year-old boy who desires occlusal correction. He underwent bilateral cleft lip and palate repair in infancy and pharyngeal flap transfer in early childhood. Examination shows Angle class III malocclusion with 12 mm of negative overjet; the SNB angle is within an acceptable range.

Which of the following is the most appropriate management?

(A) Le Fort I maxillary advancement
(B) Le Fort I maxillary advancement and genioplasty
(C) Maxillary distraction osteogenesis
(D) Maxillary and mandibular distraction osteogenesis
(E) Maxillary distraction osteogenesis and bilateral sagittal split-ramus osteotomy with mandibular setback

A

The correct response is Option C.

In patients who have severe deficiencies of the midface occurring secondary to cleft lip and palate, traditional orthognathic and orthodontic approaches are often ineffective. This patient with a bilateral cleft lip and palate has 12 mm of negative overjet following pharyngeal reconstruction with a posterior pharyngeal flap. These factors, as well as other complications seen in similar patients, including absence of maxillary and alveolar bone, scarring, and residual fistulas, can make reconstruction problematic and predispose these patients to surgical relapse. Therefore, newer procedures such as maxillary distraction osteogenesis are most appropriate for correction of the midface deficiency. This technique will expand the soft tissues and bones of the midface and palate in a single-stage procedure, correcting the malocclusion and leaving the mandible untouched.

Le Fort I maxillary advancement of more than 10 mm is a technically challenging, unpredictable procedure that would not correct this patient’s malocclusion. In addition, the posterior pharyngeal flap would have to be taken down before surgery, and a certain amount of relapse would be seen.

Although it is technically possible, a combined Le Fort I maxillary advancement and genioplasty procedure is not the first choice for this patient because the mandible and SNB angle are normal. For the same reason, mandibular distraction and/or setback are not necessary. Skeletal reduction procedures, such as the bilateral sagittal split-ramus osteotomy, are not recommended when maxillary distraction is available.

How well did you know this?
1
Not at all
2
3
4
5
Perfectly
9
Q

A 34-year-old woman comes to the office because she would like to improve the appearance of her face. She recently completed orthodontic therapy with lingual braces. When she smiles, no upper incisal show is noted. Occlusion is Angle class I. Which of the following is the most appropriate management?

A ) Cosmetic dental laminates
B ) Horizontal excision of the upper lip
C ) Mandibular osteotomy and advancement of the mandible with a genioplasty
D ) Maxillary osteotomy with vertical lengthening of the maxilla
E ) Vertical shortening of the upper lip using a scar hidden under the nostril

A

The correct response is Option D.

This case describes an adult with vertical maxillary deficiency resulting in inadequate upper incisal show. This is corrected with a maxillary osteotomy and vertical lengthening while maintaining the occlusive relationship. Excision of the upper lip is not the best solution of the patient described with vertical maxillary deficiency. A mandibular advancement should not be recommended as cephalometric evaluation is noted to be normal. Shortening the upper lip is not recommended for vertical maxillary deficiency.

How well did you know this?
1
Not at all
2
3
4
5
Perfectly
10
Q

A 41-year-old man comes to the emergency department because he is unable to close his mouth after yawning. He reports pain in the jaw. Which of the following is the most appropriate initial treatment?

A ) Arthroplasty
B ) Closed reduction during sedation
C ) Eminectomy
D ) Injection of botulinum toxin type A
E ) Intra-articular sclerosing

A

The correct response is Option B.

This patient has an acute anterior dislocation of his temporomandibular joint. Anterior dislocations are usually secondary to an interruption in the normal sequence of muscle action when the mouth closes from extreme opening. The masseter and temporalis muscles elevate the mandible before the lateral pterygoid muscle relaxes resulting in the mandibular condyle being pulled anterior to the bony eminence and out of the temporal fossa. Spasm of the masseter, temporalis, and pterygoid muscles causes trismus and keeps the condyle from returning into the glenoid fossa. Dislocations can be both unilateral and bilateral.

The most appropriate initial treatment is attempted closed reduction. Local anesthesia or sedation can help relax the muscles that are in spasm. Reduction involves downward and posterior movement of the mandible.

All other choices are options that have been tried with variable success to prevent chronic, recurrent temporomandibular joint dislocation. Arthroplasty or eminoplasty refers to augmentation of the articular eminence with a bone graft or an alloplastic material, or even titanium hardware. In contrast to the eminectomy, an eminoplasty seeks to confine the condyle to the glenoid fossa.

Eminectomy involves reducing or removing the articular eminence, which is the anterior wall of the glenoid fossa, surgically so that spontaneous reduction is possible.

Injection of botulinum toxin type A has been suggested as a treatment. The theoretic mechanism of action is relaxation of the masseter and temporalis muscles, allowing spontaneous reduction.

Intra-articular injection of a sclerosing agent, such as alcohol, usually followed by a period of interdental fixation has been described but has fallen out of favor due to lack of proven long-term efficacy. It was thought to be a noninvasive way of preventing the mandible from opening excessively wide and allowing dislocation of the condyle from the glenoid fossa by inducing fibrosis of the temporomandibular joint.

How well did you know this?
1
Not at all
2
3
4
5
Perfectly
11
Q

In a 30-year-old woman who is undergoing evaluation prior to orthognathic surgery, cephalometric analysis shows a decreased SNB angle and a normal SNA angle. These findings are most consistent with

(A) mandibular protrusion
(B) mandibular pseudoprognathism
(C) mandibular retrusion
(D) maxillary protrusion
(E) maxillary protrusion and mandibular retrusion

A

The correct response is Option C.

This patient’s findings are most consistent with mandibular retrusion, which is defined as a normal SNA (sella-nasion-point A) angle combined with a decreased SNB (sella-nasion-point B) angle on cephalometric analysis. The SNA angle measures the position of point A (anterior maxilla) relative to the anterior cranial base (SN); a normal SNA angle is defined as 82 degrees x 3 degrees. Patients with maxillary protrusion have an increased SNA angle, while patients with maxillary retrusion have a decreased SNA angle. In contrast, the SNB angle measures the position of point B (anterior mandible) relative to the anterior cranial base (SN); a normal SNB angle is defined as 80 degrees x 3 degrees. It is increased in patients with mandibular protrusion and decreased in patients with mandibular retrusion.

The Landes angle, which is formed by the Frankfort horizontal line and the nasion to point A (N-A) plane, is sometimes used instead of the SNA angle because of its greater reliability. A normal Landes angle is measured at 88 degrees x 3 degrees.

The ANB angle measures the position of point A to point B relative to the anterior cranial base; the angle is positive when point A lies anterior to point B. Patients with maxillary protrusion, mandibular protrusion, or a combination of both will have a markedly increased ANB angle. A decreased ANB angle can be seen in patients who have maxillary retrusion, mandibular protrusion, or a combination of both. In patients with mandibular pseudoprognathism, the ANB angle is normal.

How well did you know this?
1
Not at all
2
3
4
5
Perfectly
12
Q

Which of the following orthognathic movements is the most unstable and prone to relapse?

(A) Mandibular advancement
(B) Mandibular narrowing
(C) Maxillary advancement
(D) Maxillary widening
(E) Sliding genioplasty

A

The correct response is Option D.

Transverse widening of the maxilla is the most unstable orthognathic movement. With this procedure, a patient may lose as much as 50% of the movement at one year after surgery. Maxillary downgrafts and mandibular setbacks are also relatively unstable procedures. Mandibular advancement, mandibular narrowing, maxillary advancement, and sliding genioplasty are all considered stable movements.

How well did you know this?
1
Not at all
2
3
4
5
Perfectly
13
Q

A 30-year-old woman comes to the office because of a 1-year history of a clicking sensation when she opens her mouth. She was involved in a motor vehicle collision in which her face struck the steering wheel 1 year ago. Physical examination shows midline dental structures without deviation. Which of the following is the most likely cause of this patient’s condition?

A ) Disruption of the lateral pterygoid muscle

B ) Foreign body within the joint space

C ) Malunion of a coronoid fracture

D ) Nonunion of a condylar fracture

E ) Subluxation of the articular disk

A

The correct response is Option E.

Motion at the temporomandibular joint (TMJ) is best appreciated by placing one €™s fingers either inside the external auditory canal or just anterior to it. The sensation of clicking when the jaw is repeatedly opened and closed is usually caused by subluxation of the articular disk. The disk normally lies centrally between the two joint spaces. Conservative treatment involves adjustment of the patient €™s bite with a splint, anti-inflammatory drugs, and physical therapy. Surgical treatment is reserved for patients who fail conservative therapy. Air within the joint space may occur following open fractures of the mandibular condyle. The presence of a foreign body within the joint space produces pain and decreased range of motion rather than clicking. Similar symptoms are also noted in patients with degenerative disease affecting the TMJ.

How well did you know this?
1
Not at all
2
3
4
5
Perfectly
14
Q

A 22-year-old woman presents with long vertical facial height, narrow constricted alar bases, and lip incompetence. She has excessive gingival and upper incisor show at rest and while smiling. Which of the following is the most appropriate treatment for correction of the deformity?

A) Advancement genioplasty
B) Le Fort I osteotomy with impaction
C) Le Fort II osteotomy
D) Orthodontic manipulation
E) Sagittal split mandibular osteotomy

A

The correct response is Option B.

This patient presents with the classic physical finding of vertical maxillary excess, also known as long face syndrome. Patients will have long vertical facial height (especially in the lower third), narrow constricted alar bases, lip incompetence with an excessive interlabial gap, and excessive gingival and upper incisor show at rest and while smiling. They may also have a retruded and vertically long chin and a retrognathic mandible. Cephalometric analysis may show increased lower anterior facial height, SNA and SNB angles that are smaller than normal, and an ANB angle that is larger than normal (greater than 3 degrees).

The surgical treatment generally entails maxillary Le Fort I osteotomy with impaction. If there are mandibular discrepancies, then genioplasty and mandibular osteotomy may be needed. Le Fort II osteotomy would not address the vertical excess. Genioplasty alone, sagittal split mandibular osteotomy alone, and orthodontic manipulation alone would not address the vertical discrepancy.

How well did you know this?
1
Not at all
2
3
4
5
Perfectly
15
Q

In patients with vertical maxillary excess undergoing Le Fort osteotomy with maxillary impaction, which of the following findings is most likely postoperatively?
A) Increased mentalis strain
B) Increased upper incisal show
C) More obtuse nasolabial angle
D) Retrogenia
E) Widened alar base

A

Correct answer is E.

Patients with vertical maxillary excess, or long face syndrome, have a narrow alar base, an obtuse nasolabial angle, and an anterior open bite. Mentalis muscle strain and labial incompetence are increased, and there is excess gingival show and exposure of the upper incisors.
Appropriate management is Le Fort I osteotomy with maxillary impaction; osseous genioplasty is also performed in some patients. These procedures will correct many of the findings associated with this condition, including decreasing the mentalis muscle strain and incisal show and creating a more acute nasolabial angle. The alar base will be widened. Le Fort I osteotomy also rotates the mandible forward and upward, resolving the retrogenia associated with long face syndrome. Postoperative lateral cephalograms will show forward autorotation of the mandible with counterclockwise rotation.

How well did you know this?
1
Not at all
2
3
4
5
Perfectly
16
Q

A 16-year-old boy who successfully underwent Le Fort III advancement with bone grafting 6 years ago because of severe maxillomandibular disharmony comes to the office due to severe malocclusion. Physical examination shows an Angle class III malocclusion and severe mid face deficiency. Which of the following is the most likely explanation for the reappearance of this patient’s condition?

A) Age during original surgery
B) Discrepancy in the growth rate of the operated mid face and the mandible
C) Lack of bony stability in the first postoperative year
D) Poor follow-up
E) Poorly performed orthognathic surgery

A

The correct response is Option B.

A recent article showed for the first time that children who underwent Le Fort III advancement had recurrence of their initial pathology due to minimal mid face sagittal growth, but with normal mandibular growth. This study displayed this, despite excellent early advancement and bony stability up to 1 year. After 5 years, the lack of mid face growth ultimately relegates these patients to at least another advancement surgery. Definitive orthognathic surgery is required following the completion of skeletal growth to improve maxillomandibular relationships and to achieve optimal occlusion. In multiple studies, the average age of children undergoing Le Fort III osteotomies was close to age 6 years and age was not an independent factor for recidivism. Finally, studies of distraction after a Le Fort III osteotomy show better advancement and may help to minimize the recurrence of pathology.

How well did you know this?
1
Not at all
2
3
4
5
Perfectly
17
Q

In a 5-year-old child, the optimal latency period for mandibular distraction is approximately how many weeks?

(A) 1
(B) 2
(C) 3
(D) 4
(E) 8

A

The correct response is Option A.

The latency period is defined as that time following the osteotomy procedure during which the cut bone surfaces begin to become bridged by initial fracture healing, just prior to initiation of distraction. Although recommendations regarding the optimal latency period for mandibular distraction have been controversial, current clinical protocols typically describe a latency period of five to seven days. However, some studies have questioned the necessity of a latency period. In experimental studies involving adult canines, premature consolidation of the tibial bone surfaces was noted following latency periods of either 14 days or 21 days. Because latency periods of 14 days or longer have been theorized to result in premature bony union, such lengthy periods are contraindicated in distraction osteogenesis. After active distraction is complete, the bone segments are held in rigid fixation until new bone mineralization occurs. This is known as the period of consolidation, which according to clinical protocols typically lasts for eight weeks.

How well did you know this?
1
Not at all
2
3
4
5
Perfectly
18
Q

A 16-year-old boy is scheduled to undergo maxillary advancement for correction of a 10-mm negative overjet of the maxillary incisors. Which of the following additional findings is associated with the greatest risk for the development of velopharyngeal incompetence?

(A) Class III malocclusion secondary to mandibular prognathism
(B) Maxillary-mandibular disharmony secondary to craniofacial microsomia
(C) Midface hypoplasia secondary to Crouzon syndrome
(D) Midface hypoplasia secondary to repaired cleft palate

A

The correct response is Option D.

Patients with midface hypoplasia secondary to repaired cleft palate are at increased risk for development of velopharyngeal incompetence, especially following maxillary (Le Fort I) advancement of greater than 10 mm. In a study of 70 patients who underwent Le Fort I advancement, the incidence of velopharyngeal incompetence was increased in those patients who had previously undergone cleft palate repair. This was particularly true in patients who, on preoperative examination, exhibited evidence of nasal air emission, nasal resonance, borderline velopharyngeal incompetence, or a combination of these findings.

In patients with Angle class III malocclusion secondary to mandibular prognathism, maxillary advancement is not the treatment of choice; instead, the skeletal anomaly is more appropriately addressed by performing mandibular setback. This procedure should not increase the patient’s risk for development of velopharyngeal incompetence.

Patients undergoing maxillary advancement for management of other conditions, such as craniofacial microsomia or Crouzon syndrome, are at much lower risk for development of velopharyngeal incompetence than those patients with a repaired cleft palate.

How well did you know this?
1
Not at all
2
3
4
5
Perfectly
19
Q

A 40-year-old man is being evaluated because of lower dental show and occasional drooling since undergoing bilateral sagittal split mandibular osteotomy and genioplasty four years ago. Lip sensation is normal. A clinical photograph is shown. Which of the following is the most likely cause of these findings?

A ) Excessive downward repositioning of the genioplasty segment
B ) Failure to reapproximate the mentalis muscle to the mandible
C ) Injury to the buccal branch of the facial nerve
D ) Injury to the inferior alveolar nerve
E ) Injury to the marginal mandibular branch of the facial nerve

A

The correct response is Option B.

The patient shown has ptosis of the lower lip, caused by failure to reapproximate the mentalis muscle to the mandible during genioplasty. Ptosis of the soft tissues of the chin, including the lip, and excessive lower dental show are the result. If the depth of the labial sulcus is sufficiently reduced, drooling may occur.

Provided the mentalis muscle is repaired, and no nerve injury occurs, downward repositioning of the genioplasty segment should not produce excessive lower dental show.

The mentalis muscle is innervated by the marginal mandibular branch of the facial nerve, which could sustain a neurapraxic injury during either bilateral sagittal split osteotomy or genioplasty. However, that injury would be unlikely to persist for four years.

The buccal branch of the facial nerve innervates the buccinator and orbicularis oris muscles. Loss of orbicularis oris function could lead to lip ptosis, but injury to the buccal branch of the facial nerve would be very unlikely in the situation described.

Injury to the inferior alveolar nerve can occur with either bilateral sagittal split osteotomy or genioplasty. The resulting paresthesia or anesthesia of the lower lip may lead to drooling but would not cause dysfunction in the mentalis muscle or lip ptosis.

How well did you know this?
1
Not at all
2
3
4
5
Perfectly
20
Q

A 22-year-old woman comes to the office because she is unhappy with the appearance of the lower third of her face. On examination, she has a class II occlusion. Lateral cephalometric evaluation shows an SNA angle of 82 degrees (N 80-84), an SNB angle of 75 degrees (N 78-80), and an ANB angle of 7 degrees. Cranial base anatomy shows no abnormalities. Which of the following orthognathic procedures is most appropriate in this patient?

A) LeFort I maxillary advancement
B) LeFort I maxillary advancement with mandibular setback
C) Maxillary impaction
D) Sagittal split mandibular osteotomy with advancement
E) Sliding genioplasty

A

The correct response is Option D.

This patient has a skeletal class II deformity with a retrognathic mandible and normal maxillary projection. The SNA angle of 82 degrees (N 80-84) indicates a normally positioned maxilla relative to the cranial base, while the SNB angle of 75 degrees (N 78-80) indicates a retrognathic mandible relative to the cranial base. The ANB angle confirms the class II deformity (>4 degrees). A mandibular sagittal split osteotomy with advancement will correct this deformity.

Maxillary impaction is used to treat vertical maxillary excess.

LeFort I maxillary advancement will worsen this patient’s deformity.

How well did you know this?
1
Not at all
2
3
4
5
Perfectly
21
Q

A 45-year-old woman with myofascial pain dysfunction has had pain in the preauricular region for the past six months. Plain radiographs of the temporomandibular joint are most likely to show which of the following?

(A) Anterior displacement of the disk
(B) Erosion of the anterior condyle
(C) Narrowing of the joint space
(D) Osteophytes of the condylar head
(E) No abnormalities

A

The correct response is Option E.

In myofascial pain dysfunction, radiographs show no abnormalities because the disorder does not usually produce discernible anatomic abnormalities in the temporomandibular joint (TMJ). Myofascial pain dysfunction is associated with preauricular pain, occasional joint clicking, restricted jaw opening, and tenderness of the masticatory muscles. Its causes are multifactorial and include bruxism, anxiety, and occlusal abnormalities. Anterior displacement of the disk of the TMJ cannot be identified on plain radiographs because the disk is composed of fibrous tissue, which can be seen only on radiographs with contrast.

How well did you know this?
1
Not at all
2
3
4
5
Perfectly
22
Q

Three weeks after undergoing bilateral maxillary advancement, a 28-year-old man has exposure of two of the four maxillary plates in the oral cavity. The maxilla appears stable. Which of the following is the most appropriate next step in management?
A) Instruction in oral hygiene and observation
B) Irrigation and debridement of the wound followed by replacement of the two exposed plates
C) Maxillomandibular fixation with heavy elastic for two weeks
D) Removal of all plates followed by intermaxillary fixation for six weeks
E) Removal of the two exposed plates only

A

Correct answer is A,

In this patient who has exposure of the internal hardware, the most appropriate management is maintenance of optimum oral hygiene. As long as oral hygiene is maintained, the oral mucosa is likely to granulate over the maxillary plates. After satisfactory bone healing has been achieved, any exposed plates can be removed.
Removal of the hardware is not indicated in a patient who underwent surgery only three weeks earlier because there is an increased risk of bony malunion. The maxilla is stable, and maxillomandibular fixation is unnecessary. The patient can continue nutrition with a soft diet without adverse sequelae.

How well did you know this?
1
Not at all
2
3
4
5
Perfectly
23
Q

A 16-year-old boy with history of cleft lip and palate comes to the clinic for management of malocclusion. Medical history includes several surgical procedures on the palate for closure of an oral nasal fistula. Intraoral evaluation shows Class III malocclusion and 15-mm negative overjet. Lateral cephalometry shows decreased SNA angle with normal SNB angle. Which of the following surgical procedures is most appropriate for correction of this deformity?
(A) Mandibular setback
(B) Le Fort I osteotomy with bilateral sagittal split osteotomy of the mandible
(C) Le Fort I advancement by distraction osteogenesis
(D) Le Fort II advancement by distraction osteogenesis
(E) Le Fort III advancement by distraction osteogenesis

A

The correct response is Option C.

For this patient, the most appropriate surgical option is a Le Fort I advancement by distraction osteogenesis because he requires correction of the malocclusion and needs a large amount of movement. Le Fort I advancement can correct the malocclusion by moving the tooth-bearing segment of the maxilla. Distraction osteogenesis allows for large movement because it stretches the soft tissue envelope gradually to accommodate the bony framework. Also, this patient=s prior surgery probably has resulted in scarring of the soft tissue of the palate. Such scarring is also an indication for distraction, which gradually stretches the scars. In contrast, an immediate large advancement is likely to be limited by scarring.

In this patient, mandibular setback alone produces too large a movement to correct the malocclusion and risks posterior placement of the base of the tongue, which could narrow the airway. In addition, it would not be aesthetically pleasing. A mandibular setback could be used as an adjunct if it were not possible to obtain the needed advancement by moving the maxilla alone.

Because this patient’s nose, orbital rim, and malar prominences are in good position, a Le Fort II or Le Fort III advancement is not indicated.

How well did you know this?
1
Not at all
2
3
4
5
Perfectly
24
Q

A 16-year-old girl who has hypoplasia of the chin and a prominent hump on the nasal dorsum is scheduled to undergo rhinoplasty and osseous genioplasty. In this patient, nerve injury during osseous genioplasty is most likely to result in which of the following complications?

(A) Inability to depress the lower lip
(B) Inability to elevate the lower lip
(C) Numbness of the ipsilateral tongue
(D) Numbness of the lower lip
(E) Oral incompetence

A

The correct response is Option D.

This 16-year-old girl who is undergoing genioplasty is at greatest risk for injury to the mental nerve, which will result in numbness of the lower lip. The mental nerve is a sensory portion of the inferior alveolar nerve and is located distally. It should be routinely identified and preserved during surgery. Patients who have damage to the mental nerve, either from injury during genioplasty or an inferior alveolar nerve block, often inadvertently bite the lip because of the loss of sensation.

An inability to depress the lower lip during facial animation occurs as a result of injury to the facial nerve. Injury to branches of the facial nerve is most likely to result in an inability to depress or elevate the lower lip and oral incompetence. Injury to the lingual nerve would result in numbness of the tongue.

How well did you know this?
1
Not at all
2
3
4
5
Perfectly
25
Q

A 23-year-old woman undergoes Le Fort I osteotomy with impaction of the maxilla because of vertical maxillary excess. The final vertical position of the maxilla is most accurately determined using which of the following studies?
A) Assessment of maxillary lip-tooth relationship
B) Cephalometric analysis of the ANB angle
C) Cephalometric analysis of the SNA angle
D) Dental model surgery
E) Panorex radiography

A

Correct answer is A.

Assessment of the maxillary lip-tooth relationship is the most accurate study in determining the final vertical position of the maxilla. This assessment can be made on a cephalometric radiograph by performing a soft-tissue and skeletal analysis and prediction tracing. Intraoperative assessment of the amount of tooth show is also important in determining the final vertical position of the maxilla. Normally, the lip-tooth relationship is 2 to 3 mm of tooth show.
Cephalometric analysis of the ANB and SNA angles are skeletal measurements. The ANB angle relates the maxilla to the mandible in the horizontal plane. The SNA angle relates the maxilla to the base of the cranium in the horizontal plane. Neither measurement assesses the vertical position of the maxilla or the soft-tissue envelope.

How well did you know this?
1
Not at all
2
3
4
5
Perfectly
26
Q

A 20-year-old woman with juvenile rheumatoid arthritis has worsening occlusion two years after undergoing sagittal split osteotomy with mandibular advancement. On examination, there is a loss of posterior facial height bilaterally and an anterior open bite. She has Angle class II malocclusion. Serial cephalometric analysis shows progressive posterior movement of the B point. Which of the following is the most likely cause of the worsening occlusion?

(A) Continued growth of the maxilla
(B) Improper intraoperative seating of the condyles in the glenoid fossae
(C) Improper preoperative and postoperative orthodontic treatment
(D) Loosening of all of the plates of the rigid internal fixation
(E) Progressive condylar resorption

A

The correct response is Option E.

Progressive condylar resorption is a late cause of open bite that occurs mainly in young women. It is associated with condylar shortening, a decrease in posterior facial height, clockwise rotation of the mandible, and Angle class II malocclusion. Slow progressive posterior movement of the point B on serial cephalometric analysis is a classic finding. The exact cause of the problem is unknown.

Poor orthodontic treatment can result in a recurrence of malocclusion in the postoperative period. However, the patient described above exhibits many of the classic findings of progressive condylar resorption, making that a more likely diagnosis.

The most likely cause of immediate postoperative open bite is improper seating of the condyles in the glenoid fossae during surgery. It is important to take the patient out of intermaxillary fixation after fixation of the osteotomies is completed to ensure that the condyles are properly seated. During this process, the occlusion and path of the opening of the mandible are checked. In a skeletally mature female, continued growth of the maxilla would be unusual. Lastly, it would be unusual for all of the plates of the rigid internal fixation to loosen.

How well did you know this?
1
Not at all
2
3
4
5
Perfectly
27
Q

A 10-year-old boy is scheduled to undergo Le Fort III osteotomy with distraction osteogenesis for advancement of the midface. Which of the following is an advantage of using a rigid external distraction device rather than an internal distraction device in this patient?

(A) Decreased operative morbidity
(B) Decreased risk for relapse following midface advancement
(C) Greater degree of advancement
(D) More rapid rates of distraction
(E) Need for fewer subsequent operative procedures

A

The correct response is Option E.

Distraction osteogenesis with external or internal distraction devices can be performed to lengthen the midface gradually in children with craniosynostosis, cleft lip and palate, hemifacial microsomia, and midface hypoplasia. Midface osteotomies must be performed in order to initiate distraction regardless of the type of device. The rigid external distraction (RED) device is most commonly used externally. Distraction with this device, rather than with an internally implanted device, is advantageous primarily because it can be removed in the office and a second operative procedure is not necessary. In contrast, an additional operative procedure is required in a patient who has an implanted internal device to expose and remove the hardware. Resorbable internal devices have been introduced recently, which may decrease the extent of operative dissection required for removal. However, a second operative procedure is still necessary to remove the metallic distraction foot plate attached to the resorbable hardware.

There are no differences in the degree of operative morbidity at the time of osteotomy with either the internal or external distraction device. However, operative morbidity following midface osteotomy for distraction osteogenesis is lower than that seen with midface osteotomy using conventional bone grafting and rigid fixation techniques. Distraction techniques using either internal or external devices do not have the potential for morbidity associated with bone graft harvest sites.

The potential for relapse of the advanced midface segment is no different with either technique following removal of the distraction device and an adequate period of consolidation.

There are no data to indicate that the results achieved with either type of device differ substantially. There is also no difference in the rate of distraction or the amount of distraction that can be achieved with either technique. Both techniques require a period of bone consolidation after distraction has been completed.

How well did you know this?
1
Not at all
2
3
4
5
Perfectly
28
Q

Which of the following is the most common complication of sagittal split osteotomy?

(A) Avascular necrosis of the proximal segment
(B) Injury to the mandibular molars
(C) Loss of lower lip sensibility
(D) Nonunion of the osteotomy site
(E) Relapse from incorrect positioning of the condyles at surgery

A

The correct response is Option C.

The most common complication of sagittal split osteotomy is loss of lower lip sensibility. Studies have shown a significant incidence of both temporary and permanent disruption of sensibility in the lower lip following this procedure.

Sagittal split osteotomy involves only minimal muscle stripping on the lateral aspect of the mandible. The blood supply to the proximal bony segment is adequate, and the incidence of avascular necrosis is minimal.

Because the roots of the mandibular molars are closer to the lingual cortex than to the buccal cortex, they are not typically injured during osteotomy.

Nonunion is an extremely rare complication of sagittal split osteotomy.

During the osteotomy procedure, the proximal segment and condyle are seated gently into the fossa, and the surgeon takes great care to avoid displacing the condyle anteriorly and downward on the temporal bone. As a result, relapse from incorrect positioning occurs only rarely.

How well did you know this?
1
Not at all
2
3
4
5
Perfectly
29
Q

A 22-year-old woman comes to the office for evaluation of an abnormal bite. On physical examination, she has an anterior open bite, and the upper teeth are not exposed with the lips in repose. Cephalometric analysis shows a nasion (N) to anterior nasal spine (ANS) distance of 45 mm (N 52–57 mm), an ANS to menton (Me) distance of 63 mm (N 63–68 mm), and an N-ANS:ANS-Me ratio of 1:1.4 (N 1:1.2). All other measurements are within the reference ranges. Which of the following is the most appropriate surgical procedure for correction of this patient’s deformity?

A) Le Fort I maxillary osteotomy with downward repositioning
B) Le Fort II osteotomy with maxillary advancement
C) Naso-orbito-maxillary osteotomy
D) Perinasal osteotomy
E) Sagittal split osteotomy with mandibular setback

A

The correct response is Option A.

Le Fort I osteotomy with downward repositioning effectively lengthens the maxilla in cases of isolated vertical maxillary hypoplasia. The maxilla is repositioned vertically in its entirety or rotated downward, depending on whether or not the hypoplasia extends to the posterior maxilla. The goal is to close the anterior open bite and to restore facial height, allowing 3 to 4 mm of upper incisor to show with lips in repose.

Perinasal osteotomy is a procedure designed to lengthen the skeletal framework of the nose. It lengthens and increases nasal projection. It is therefore a suitable procedure for patients with nasomaxillary hypoplasia and a foreshortened nose, but with normal dental occlusion and facial height. It does not correct maxillary height or change the dental relationships.

Naso-orbito-maxillary osteotomy is a step beyond perinasal osteotomy, in that it corrects both the foreshortened and retruded nasal framework and maxillary hypoplasia horizontally and vertically. The entire osteotomized segment includes the central section of the maxilla from nasion to teeth, and from one internal orbital rim to the other. It can therefore close an anterior open bite when vertical maxillary insufficiency is a component of the deformity in addition to a retruded nasomaxillary complex. However, it would most likely shift the occlusion into class II if there were not also a horizontal deficiency of the maxilla. Therefore, it is not an appropriate procedure for the patient in the vignette because it would alter the naso-orbital region unnecessarily, and possibly cause a new deformity or abnormal relationship in this otherwise isolated vertical maxillary deficiency. The indications for a or a naso-orbito-maxillary osteotomy would overlap those for a Le Fort II osteotomy.

Le Fort II osteotomy is indicated for nasomaxillary hypoplasia with a recessed maxilla and class III malocclusion. This is frequently noted in patients with a history of cleft lip and palate. The same discussion used for the naso-orbito-maxillary osteotomy would apply here as well.

Sagittal split osteotomy is a procedure that modifies the mandible, permitting setback or advancement of the mandibular dentition when the cause of the malocclusion is mandibular hypoplasia or overdevelopment. It has no effect on the maxilla.

How well did you know this?
1
Not at all
2
3
4
5
Perfectly
30
Q

A 22-year-old man desires aesthetic enhancement of the chin. Examination of the chin shows horizontal (sagittal) deficiency and vertical excess. Which of the following surgical procedures should be performed in this patient?

(A) Alloplastic chin augmentation
(B) Interpositional genioplasty
(C) Jumping genioplasty
(D) Reduction genioplasty

A

The correct response is Option C.

A jumping genioplasty can performed to correct both horizontal (sagittal) deficiency and vertical excess. Preoperative evaluation of patients with chin abnormalities should involve analysis of the horizontal and vertical (ie, superoinferior) dimensions; chin symmetry should also be noted. Patients can have a combination of deficiency and/or excess in both the horizontal and vertical dimensions. In order to correct the findings seen in this patient, horizontal osteotomy is performed first, and then the caudal mandibular osteotomized segment is placed in front of the mandible.

Alloplastic augmentation is typically reserved for older persons with minimal pure horizontal chin deficiencies, and osseous genioplasties are performed in younger patients with more pronounced anomalies. Interpositional genioplasty, which is indicated for correction of vertical deficiency alone, is accomplished by performing a horizontal osteotomy with interpositional grafting using bone or a bone substitute. Reduction genioplasty, which involves wedge ostectomy followed by securing of the caudal segment to the mandible, is used for correction of vertical excess alone.

How well did you know this?
1
Not at all
2
3
4
5
Perfectly
31
Q

Which of the following is the most common cause of temporomandibular joint ankylosis?

A) Bruxism
B) Congenital anomaly
C) Infection
D) Radiation
E) Trauma

A

The correct response is Option E.

The most common cause of temporomandibular joint (TMJ) ankylosis is trauma. It usually occurs after untreated or inadequately treated mandibular fractures. Damage to the articular surface of the TMJ is the most common factor seen. In children, this can lead to growth disturbances ultimately requiring orthognathic surgery. Otherwise, joint replacement and repair may be indicated in adults. In the antibiotic era, infection is a rare cause. Congenital anomalies, bruxism, and radiation are less common.

How well did you know this?
1
Not at all
2
3
4
5
Perfectly
32
Q

The Frankfort horizontal line is formed by connecting which of the following two points on a standard cephalogram?

(A) A point and B point

(B) B point and menton

(C) Nasion and porion

(D) Orbitale and porion

(E) Sella and orbitale

A

The correct response is Option D.

A line joining the porion and the orbitale forms the Frankfort horizontal line (see cephalogram below). The porion is the superior aspect of the external auditory meatus. The orbitale is the most inferior point around the orbital rim. The Frankfort horizontal line is used to determine other reference measurements in the facial skeleton, such as the facial plane angle. The A point is the most posterior point of the anterior surface of the maxilla. The B point is the most posterior point on the anterior surface of the mandible. The menton is the most inferior point on the lower border of the mandible. The nasion is the junction of the frontal bone and the nasal bones. The sella is the midportion of the sella turcica. Lines and angles other than the Frankfort horizontal line utilize these points to determine anomalous cranial skeletal structure.

How well did you know this?
1
Not at all
2
3
4
5
Perfectly
33
Q

A 37-year-old woman is undergoing evaluation because of intermittent clicking of the right temporomandibular joint (TMJ). She has no pain or crepitus of the joint. Interincisal opening is 40 mm. MRI shows a nonreducing articular disk within the right TMJ. Which of the following is the most appropriate management?

(A) Observation
(B) Intracapsular repositioning of the disk
(C) Intracapsular repositioning of the disk and reduction of the articular eminence
(D) Removal of the disk and placement of an interpositional temporalis fascia flap

A

The correct response is Option A.

Conservative management is most appropriate for this patient who has episodic clicking, no pain, and a normal interincisal opening distance. Operative correction is indicated only for internal derangement of the temporomandibular joint associated with congenital anomalies, neoplasia, previous trauma to the joint, chronic pain, or trismus resulting in functional limitation.

Intracapsular repositioning of the disk, removal of the disk, and placement of a temporalis fascia flap are options for those patients with internal derangement who are surgical candidates. Surgical reduction of the articular eminence, or eminectomy, is indicated in patients who have symptomatic open locking of the mandible.

How well did you know this?
1
Not at all
2
3
4
5
Perfectly
34
Q

Which of the following percentages best represents the incidence of paresthesia of the lower lip immediately after bilateral sagittal split osteotomy?
(A) 10%
(B) 30%
(C) 50%
(D) 70%
(E) 90%

A

The correct response is Option E.

Paresthesia of the lower lip is the most common immediate postoperative finding following a bilateral sagittal split osteotomy. It is generally bilateral and is due to neurapraxia resulting from stretch and compression of the inferior alveolar nerve as the mandible is mobilized and fixed into its new position. Studies have shown that the incidence of this finding ranges from 85% to 97% in the immediate postoperative period. In one study, 55% of the patients reported some degree of paresthesia at one month, which was further reduced to 12.5% at one year. The older the patient, the more protracted the sensory deficit.

How well did you know this?
1
Not at all
2
3
4
5
Perfectly
35
Q

A 7-year-old boy with Pfeiffer syndrome is brought to the office because of snoring that has worsened progressively for the past 6 months. Treatment with tonsillectomy and adenoidectomy failed to correct the obstruction. Physical examination shows moderate proptosis, Angle class III malocclusion, and inadequate malar projection. Nasendoscopy shows pharyngeal obstruction by the soft palate. Which of the following is the most appropriate management?

A ) Le Fort I osteotomy and advancement
B ) Le Fort II osteotomy and advancement
C ) Le Fort III osteotomy and advancement
D ) Mandibular setback with glossopexy and tongue-lip adhesion
E ) Mandibular setback with vertical ramus osteotomies

A

The correct response is Option C.

In a patient with Pfeiffer syndrome who is found to have worsening nasopharyngeal airway obstruction, mid face advancement is indicated to obviate a tracheostomy. Until the adult dentition has erupted, a Le Fort I osteotomy is contraindicated to avoid injuring the developing teeth in the maxilla. A Le Fort II osteotomy will fail to advance the deficient malar processes. A mandibular setback, by any means, will worsen the airway.

How well did you know this?
1
Not at all
2
3
4
5
Perfectly
36
Q

A 33-year-old woman presents to the office for evaluation of facial pain. Physical examination shows that she has facial pain, temporomandibular joint clicking, and an anterior open bite. Which of the following is the most common physical examination finding of temporomandibular joint dysfunction?

A) Dental pain
B) Jaw deviation with mouth opening
C) Malocclusion
D) Masseter hypertrophy
E) Pain on palpation of the muscles of mastication

A

The correct response is Option E.

The most common age at presentation for temporomandibular disorder (TMD) is 20 to 40 years, and it is more common in women than men. Occlusal abnormalities, such as open bite, have not been shown to be a major cause of TMD, as is evidenced by the lack of response to occlusal correction therapy. Pain on palpation of the muscles of mastication is the most consistent clinical sign seen with TMD. Localized dental pain is not often a feature of TMD. Masseter hypertrophy is associated with bruxism.

How well did you know this?
1
Not at all
2
3
4
5
Perfectly
37
Q

A 12-year-old girl is scheduled to undergo surgically assisted maxillary expansion for correction of transverse maxillary deficiency. During this subtotal Le Fort I procedure, completion of each of the following osteotomies is appropriate EXCEPT
(A) lateral nasal walls bilaterally
(B) anterior and lateral antral walls bilaterally
(C) pterygoid plates bilaterally
(D) palatal midline
(E) nasal septum

A

The correct response is Option A.

Complete osteotomy of the thin lateral nasal walls is unnecessary. This structure offers little resistance to transverse expansion. Additionally, the lateral nasal walls help maintain the spatial relationship of the mobilized maxilla.

Completion osteotomies of the anterior antral walls, the lateral antral walls, and the pterygoid plates bilaterally as well as the midpalatal suture and the nasal septum are necessary for unhindered symmetric expansion of the maxillary halves.

How well did you know this?
1
Not at all
2
3
4
5
Perfectly
38
Q

A 20-year-old woman comes to the office for consultation regarding malocclusion. Cephalometric analysis shows SNA angle of 83 degrees (normal = 82 ± 3 degrees) and SNB angle of 85 degrees (normal = 80 ± 3 degrees). Which of the following is the most likely cause of this patient’s deformity?
A) Mandibular prognathism
B) Mandibular retrognathism
C) Maxillary retrusion
D) Retrogenia
E) Vertical maxillary excess

A

Correct answer is A.

A normal SNA (sella nasion point A) angle is defined as 82 ± 3 degrees and measures the position of point A (anterior maxilla) relative to the anterior cranial base (SN). A normal SNB (sella nasion point B) angle is defined as 80 ± 3 degrees and measures the position of point B (anterior mandible) relative to the cranial base (SN). The most common cause of a normal SNA with an increased SNB is mandibular prognathism. Mandibular retrognathism would have a normal SNA with a decreased SNB. Maxillary retrusion is associated with a decreased SNA and a normal SNB. Vertical maxillary excess alone would not change the SNB but describes a patient with long face syndrome.

How well did you know this?
1
Not at all
2
3
4
5
Perfectly
39
Q

A 20-year-old woman comes to the office for consultation regarding malocclusion. Cephalometric analysis shows SNA angle of 83 degrees (normal = 82 ± 3 degrees) and SNB angle of 85 degrees (normal = 80 ± 3 degrees). Which of the following is the most likely cause of this patient’s deformity?

(A) Mandibular prognathism

(B) Mandibular retrognathism

(C) Maxillary retrusion

(D) Retrogenia

(E) Vertical maxillary excess

A

The correct response is Option A.

A normal SNA (sella €‘nasion €‘point A) angle is defined as 82 ± 3 degrees and measures the position of point A (anterior maxilla) relative to the anterior cranial base (SN). A normal SNB (sella €‘nasion €‘point B) angle is defined as 80 ± 3 degrees and measures the position of point B (anterior mandible) relative to the cranial base (SN). The most common cause of a normal SNA with an increased SNB is mandibular prognathism. Mandibular retrognathism would have a normal SNA with a decreased SNB. Maxillary retrusion is associated with a decreased SNA and a normal SNB. Vertical maxillary excess alone would not change the SNB but describes a patient with long face syndrome.

How well did you know this?
1
Not at all
2
3
4
5
Perfectly
40
Q

The percentage of patients who have numbness in the distribution of the mental nerve one year after undergoing sagittal split osteotomy is closest to…
A) 0%
B) 10%
C) 30%
D) 60%
E) 80%

A

Correct answer is B.

According to the results of several studies, the risk for permanent damage to the inferior alveolar nerve during sagittal split osteotomy is 5% to 10%. The inferior alveolar nerve exits from the mental foramen to become the mental nerve, and the incidence of permanent sensory disturbance in the distribution of the mental nerve is similar to the incidence in the inferior alveolar nerve.

How well did you know this?
1
Not at all
2
3
4
5
Perfectly
41
Q

A 32-year-old man is undergoing evaluation because he has temporal headaches and a sensation of “sand in the jaw” when he eats. He sustained trauma to the face while playing football in college. On current physical examination, there is reciprocal clicking and transient locking of the jaw during opening and closing movements. MRI shows anterior malpositioning of the meniscus and posterosuperior displacement of the condyle.

These findings are most consistent with which of the following?

(A) Ankylosis of the temporomandibular joint
(B) Avascular necrosis of the condylar head of the mandible
(C) Dislocation of the temporomandibular joint
(D) Internal derangement of the temporomandibular joint
(E) Myofascial pain dysfunction syndrome

A

The correct response is Option D.

The findings in this patient are most consistent with internal derangement of the temporomandibular joint (TMJ), which is defined as an abnormal relationship between the articular disk and mandibular condyle. This condition is typically associated with anterior displacement of the meniscus and often with posterosuperior malpositioning of the condyle. The retromeniscal pad may be damaged and/or disrupted. Affected patients usually have preauricular pain and clicking of the joint, as well as other, less specific symptoms such as headache and aching pain in the ear or neck. A history of trauma or previous orthodontic treatment may be associated.

Ankylosis of the TMJ can result from trauma, infection, juvenile rheumatoid arthritis, or other conditions. In patients with TMJ ankylosis, destruction of the articular disk and joint elements occurs, resulting in fibrosis, narrowing of the joint space, and bony fusion.

Although avascular necrosis is rarely seen within the mandibular condyle, it may occur as a result of trauma or devascularization at the time of TMJ surgery. Affected patients have pain and limited jaw motion; MRI will show devascularization of the condyle.

Acute TMJ dislocation occurs following anterior extension of the condyle beyond the eminence; this condition occurs as a result of joint hypermobility secondary to either trauma or an excessively large mouth opening. Although spontaneous relocation typically follows, manual reduction under anesthesia may be required.

Patients with myofascial pain dysfunction have short, sudden episodes of aching pain in the jaw associated with stress; this condition is often referred to as masticatory muscle spasm secondary to bruxism. Malocclusion associated with long-term microtrauma to the joint (as seen in patients with bruxism) is thought to result in spasm of the lateral pterygoid or deep posterior masseter muscle, which is then further aggravated by episodes of anxiety and stress.

How well did you know this?
1
Not at all
2
3
4
5
Perfectly
42
Q

Alloplastic chin augmentation is most appropriate for a patient with which of the following findings?

A

The correct response is Option D.

Because the indications for alloplastic chin augmentation are limited, the surgeon should carefully consider the skeletal deficiencies of each patient before considering this procedure. Alloplastic augmentation is most appropriate for a patient who has a minimal sagittal deficiency of the lower face, a shallow labiomental fold, and symmetry and normal height of the lower face.

Alloplastic chin implants are not appropriate for patients with chin asymmetry. Because a chin implant is always placed over the anterior symphysis, it cannot be used to correct abnormalities in facial height. Chin implantation will further accentuate any preexisting deep labiomental folds, resulting in an unnatural, “operated” appearance. Osseous genioplasties are more appropriate instead in those patients who require adjustments in the height of the lower face, or in patients who have a deep labiomental fold and require genial advancement. The vertical height of the chin can be elongated during advancement of the chin to prevent further deepening of the labiomental fold.

How well did you know this?
1
Not at all
2
3
4
5
Perfectly
43
Q

A 19-year-old man comes to the office for evaluation of a large lower jaw. Physical examination shows an Angle class III malocclusion. Which of the following is the most likely finding on cephalometric evaluation?

A) ANB angle is more acute than normal
B) SNA angle is more obtuse than normal
C) SNB angle is more obtuse than normal
D) SNO angle is more acute than normal

A

The correct response is Option C.

In a patient with mandibular prognathism (ie, projecting mandible), the SNB angle is more likely to be larger or more obtuse than normal. On a lateral cephalogram, the SNB is formed by the angle drawn between the sella, nasion, and B point (supramentale) of the mandible.

The ANB angle describes the position of the mandible relative to the maxilla. In the scenario described, the ANB angle would be more acute than normal.

The SNA angle describes the sagittal position of the maxilla and would be expected to be normal in the patient described.

The SNO angle describes the relationship between the inferior orbital rim and the skull base.

An image is shown.

How well did you know this?
1
Not at all
2
3
4
5
Perfectly
44
Q

An 18-year-old woman is referred for evaluation of lower facial asymmetry. Examination shows the mandibular dental midline 6 mm to the left of the midsagittal plane, and the chin point and maxilla are coincident with the midsagittal plane. Which of the following procedures is most appropriate to correct the lower facial asymmetry?

(A) Le Fort I osteotomy with midline shift to the left and mandibular sagittal osteotomy with shift to the right

(B) Mandibular sagittal osteotomy with midline shift to the left

(C) Mandibular sagittal osteotomy with midline shift to the right

(D) Mandibular sagittal osteotomy with midline shift to the left and sliding genioplasty to the right

(E) Mandibular sagittal osteotomy with midline shift to the right and sliding genioplasty to the left

A

The correct response is Option E.

In assessing the orthognathic patient preoperatively, it is important to recognize all asymmetries and to be aware of potential asymmetries that may result from planned procedures. Combined osteotomies of the mandible and chin are indicated to bring the mandibular dental midline and the bony chin midline in line with the midsagittal plane in the patient described. To correct the jaw deformity, the mandibular dental midline would have to move to the right. The chin point would also move to the right and would therefore require an osteotomy and left shift.

Combining a Le Fort I and mandibular sagittal osteotomies would result in coincident dental midlines, but neither would be aligned with the midsagittal plane.

Correcting the dental midline alone with mandibular sagittal osteotomies would create a genial asymmetry that was not present preoperatively.

How well did you know this?
1
Not at all
2
3
4
5
Perfectly
45
Q

A 16-year-old boy with history of cleft lip and palate comes to the clinic for management of malocclusion. Medical history includes several surgical procedures on the palate for closure of an oral nasal fistula. Intraoral evaluation shows Class III malocclusion and 15-mm negative overjet. Lateral cephalometry shows decreased SNA angle with normal SNB angle. Which of the following surgical procedures is most appropriate for correction of this deformity?
(A) Mandibular setback
(B) Le Fort I osteotomy with bilateral sagittal split osteotomy of the mandible
(C) Le Fort I advancement by distraction osteogenesis
(D) Le Fort II advancement by distraction osteogenesis
(E) Le Fort III advancement by distraction osteogenesis

A

The correct response is Option C.

For this patient, the most appropriate surgical option is a Le Fort I advancement by distraction osteogenesis because he requires correction of the malocclusion and needs a large amount of movement. Le Fort I advancement can correct the malocclusion by moving the tooth-bearing segment of the maxilla. Distraction osteogenesis allows for large movement because it stretches the soft tissue envelope gradually to accommodate the bony framework. Also, this patient=s prior surgery probably has resulted in scarring of the soft tissue of the palate. Such scarring is also an indication for distraction, which gradually stretches the scars. In contrast, an immediate large advancement is likely to be limited by scarring.

In this patient, mandibular setback alone produces too large a movement to correct the malocclusion and risks posterior placement of the base of the tongue, which could narrow the airway. In addition, it would not be aesthetically pleasing. A mandibular setback could be used as an adjunct if it were not possible to obtain the needed advancement by moving the maxilla alone.

Because this patient’s nose, orbital rim, and malar prominences are in good position, a Le Fort II or Le Fort III advancement is not indicated.

How well did you know this?
1
Not at all
2
3
4
5
Perfectly
46
Q

In an 18-year-old man with Angle class III malocclusion, cephalometric analysis shows a decreased SNA angle and a normal SNB angle. Which of the following is the most likely cause of these findings?

(A) Mandibular prognathism
(B) Mandibular retrognathism
(C) Maxillary retrusion
(D) Vertical maxillary excess

A

The correct response is Option C.

Maxillary retrusion is the most common cause of a decreased SNA (sella-nasion-point A) angle combined with a normal SNB (sella-nasion-point B) angle. The SNA angle measures the position of point A (anterior maxilla) relative to the anterior cranial base (SN); a normal SNA angle is defined as 82 degrees  3 degrees. Patients with maxillary protrusion have an increased SNA angle, while patients with maxillary retrusion have a decreased SNA angle. In contrast, the SNB angle measures the position of point B (anterior mandible) relative to the anterior cranial base (SN); a normal SNB angle is defined as 80 degrees  3 degrees. It is increased in patients with mandibular protrusion and decreased in patients with mandibular retrusion.
This patient has Angle class III malocclusion, in which the mandible is abnormally protrusive relative to the maxilla, or the maxilla is retrusive relative to the mandible. This patient has a decreased SNA angle and a normal SNB angle, as stated above; therefore, the mandibular position is normal while the maxillary position is deficient.
How well did you know this?
1
Not at all
2
3
4
5
Perfectly
47
Q

An 18 year old man is undergoing presurgical orthodontic therapy for a dentofacial skeletal deformity. A current photograph is shown. Orthognathic surgery is planned. Repositioning of the maxilla during this procedure may result in each of the following changes EXCEPT

(A) cephalic rotation of the nasal tip

(B) decreased width of the nasal alar base

(C) flattening of the upper lip

(D) obstruction of the nasal airway

(E) reduced exposure of the vermilion

A

The correct response is Option B.

The patient shown has vertical maxillary excess as well as mandibular retrognathia. Le Fort I osteotomy with vertical impaction of the maxilla is necessary. Superior repositioning of the maxilla may result in several undesirable changes in nasolabial aesthetics, including increased width of the nasal alar base. Decrease in the width of the nasal alar base will not result from this procedure. Other possible nasal changes include cephalic rotation and increased projection of the nasal tip, reduction in the nasolabial angle, as well as flattening and shortening of the upper lip with resultant loss of visible vermilion.

How well did you know this?
1
Not at all
2
3
4
5
Perfectly
48
Q

A 50-year-old man is evaluated for a 6-month history of clicking of the left temporomandibular joint and pain with joint movement. Physical examination shows an interincisal opening of 20 mm and recurrent locking in the open position; no signs of infection or ankylosis are noted. Which of the following is the most appropriate treatment?

A ) Botulinum toxin type A injection
B ) Intracapsular disk repositioning and reduction of the articular eminence
C ) Removal of disk and placement of an interpositional temporalis fascia flap
D ) Temporomandibular joint replacement
E ) Observation

A

The correct response is Option B.

Surgical reduction of the articular eminence is indicated for patients who have symptomatic open locking of the mandible. Surgical options for symptomatic patients and secondary functional limitations with internal joint abnormalities on MRI include: intracapsular disk repositioning; discectomy; and an interpositional temporalis fascia flap. Botulinum toxin type A is not an approved use in this clinical setting. Discectomy is appropriate as a salvage procedure. Temporomandibular joint replacement is rarely warranted unless signs of infection, as part of cancer resection, or severe ankylosis are noted. Conservative management is appropriate in cases with no functional abnormalities.

How well did you know this?
1
Not at all
2
3
4
5
Perfectly
49
Q

An 18-year-old man with a history of cleft lip and palate repair is planning to undergo orthognathic surgery to correct his malocclusion. When counseling the patient regarding complications, which of the following is the most common complication after orthognathic surgery?

A) Dental injury
B) Hardware failure
C) Hemorrhage
D) Infection
E) Nerve injury

A

The correct response is Option E.

Large-scale studies involving hundreds to even thousands of patients have demonstrated the safety and risks associated with orthognathic surgery. Although all of the complications listed in the options have been described during orthognathic surgery, the most common complication is nerve injury ranging from 12.1 to 19%. Infection has been reported from 2 to 3.4%. Hardware failure occurs in up to 2.5% of cases, while dental injuries range from 0.14 to 5%. Significant hemorrhage occurs in 0.5 to 1.4% of cases. Additional complications associated with orthognathic surgery include postoperative temporomandibular joint disorders/condylar disorder, scar problems, and necessity of secondary surgery.

How well did you know this?
1
Not at all
2
3
4
5
Perfectly
50
Q

A 12-year-old girl is scheduled to undergo surgically assisted maxillary expansion for correction of transverse maxillary deficiency. During this subtotal Le Fort I procedure, completion of each of the following osteotomies is appropriate EXCEPT
(A) lateral nasal walls bilaterally
(B) anterior and lateral antral walls bilaterally
(C) pterygoid plates bilaterally
(D) palatal midline
(E) nasal septum

A

The correct response is Option A.

Complete osteotomy of the thin lateral nasal walls is unnecessary. This structure offers little resistance to transverse expansion. Additionally, the lateral nasal walls help maintain the spatial relationship of the mobilized maxilla.

Completion osteotomies of the anterior antral walls, the lateral antral walls, and the pterygoid plates bilaterally as well as the midpalatal suture and the nasal septum are necessary for unhindered symmetric expansion of the maxillary halves.

How well did you know this?
1
Not at all
2
3
4
5
Perfectly
51
Q

A 21-year-old man undergoes discectomy for the treatment of temporomandibular joint internal derangement. Which of the following long-term complications is most likely for this patient?

A) Ankylosis
B) Condylar remodeling
C) Facial (VII) nerve injury
D) Frey syndrome

A

The correct response is Option B.

The temporomandibular joint (TMJ) is described as a ginglymoarthrodial joint since it is both a ginglymus (hinging joint) and an arthrodial (sliding) joint. A superior joint space is separated from an inferior joint space by a disc. The arterial supply to the TMJ is provided by the branches of the superficial temporal branch of the external carotid, in addition to the deep auricular, ascending pharyngeal, and maxillary arteries. Nervous innervation is provided by the auriculotemporal and masseteric branches of the mandibular nerve (cranial nerve V3). Resection of the disc is a treatment for internal derangement that is not responsive to conservative measures. The most common long-term complication of discectomy is remodeling of the mandibular condyle. While Frey syndrome, facial (VII) nerve injury, and ankylosis are possible, they are less common than remodeling.

How well did you know this?
1
Not at all
2
3
4
5
Perfectly
52
Q

A 17-year-old girl with Marfan syndrome comes to the office for an orthognathic evaluation. Intraoral examination shows a bilateral posterior lingual crossbite. Which of the following is the most appropriate management?

(A) Le Fort I osteotomy with palatal expansion

(B) Le Fort I osteotomy with palatal narrowing

(C) Mandibular osteotomy with narrowing

(D) Mandibular osteotomy with widening

A

The correct response is Option A.

Patients with Marfan syndrome typically have a high-arched and narrow palate resulting in a transversely constricted maxilla. Le Fort I osteotomy with palatal expansion is an appropriate surgical option for correcting the skeletal facial disharmony in the patient described.

Palatal narrowing would further worsen the skeletal problem.

Mandibular osteotomies are rarely performed to correct malocclusions based in the maxilla. However, if performed, they would generally be applied to narrow the mandible anteriorly.

How well did you know this?
1
Not at all
2
3
4
5
Perfectly
53
Q

The percentage of patients who have numbness in the distribution of the mental nerve one year after undergoing sagittal split osteotomy is closest to

(A) 0%
(B) 10%
(C) 30%
(D) 60%
(E) 80%

A

The correct response is Option B.

According to the results of several studies, the risk for permanent damage to the inferior alveolar nerve during sagittal split osteotomy is 5% to 10%. The inferior alveolar nerve exits from the mental foramen to become the mental nerve, and the incidence of permanent sensory disturbance in the distribution of the mental nerve is similar to the incidence in the inferior alveolar nerve.

How well did you know this?
1
Not at all
2
3
4
5
Perfectly
54
Q

A 24-year-old woman is referred to the office by her orthodontist for evaluation of facial disharmony. The following angles are obtained on cephalometric analysis: SNA 70 (normal = 81.2) SNB 77 (normal = 77.3) SN-pogonion 87 (normal = 80) Which of the following procedures is most effective to achieve facial symmetry in this patient?
A) Le Fort I advancement and advancement genioplasty
B) Le Fort I advancement and setback genioplasty
C) Sagittal split mandibular advancement
D) Sagittal split mandibular setback
E) Vertical ramus osteotomy and setback

A

Correct answer is B

The cephalogram is a standardized radiograph used for analysis of facial disharmony and asymmetry. Labeled landmarks help establish the relationship of the maxilla, mandible, and skull base to other facial structures. The sella (S) point marks the center of the hypophyseal fossa. The nasion (N) is the junction of the nasal and frontal bones at the most posterior point of the curve of the nose. The A point marks the innermost curvature from the maxillary anterior nasal spine to the alveolar process. The angle created by these points (SNA) establishes the maxillary position in relation to the skull base. The B point is located at the innermost curvature from the chin to the alveolar process of the mandible. The SNB angle similarly establishes the mandibular relationship to the skull base.
The prominence of the chin is often an important consideration in orthognathic surgery. The pogonion (Pg) is the most anterior chin point. The SNPg angle is representative of the degree of chin prominence relative to the SNB (mandible position).

Normal angle values are given in the text. This patient has a relatively retrusive maxilla and a normally positioned mandible. This would represent a class III relation. The pogonion is anteriorly displaced in relation to both the mandible and the maxilla.

Therefore, to establish better facial relationships, the maxilla should be advanced at the Le Fort I level and the chin setback via a genioplasty. The mandible does not need to be moved.

How well did you know this?
1
Not at all
2
3
4
5
Perfectly
55
Q

Which of the following reference points are used in cephalometric analysis of the mandible?

(A) Glabella and subnasale

(B) Nasion and pogonion

(C) Orbitale and porion

(D) Orbitale and sella

(E) Sella and A point

A

The correct response is Option B.

€œNasion and pogonion € is the only option that has at least one point of reference on the mandible. A line drawn between the nasion and pogonion may be referred to as the facial axis plane, and it may be related to any of several other intersecting lines, such as the Frankfort horizontal (orbitale to porion) line, to evaluate the mandible. The sella, A point, and palatal plane are used to evaluate the position of the maxilla.

56
Q

A 35-year-old man comes to the emergency department because he has been unable to close his mouth since hearing a popping sound in his jaw during a wide yawn. Which of the following is the most appropriate initial management?

(A) Injection of a corticosteroid
(B) Attempted closed reduction in the emergency department under sedation
(C) Disk plication
(D) Eminectomy

A

The correct response is Option B.

This 35-year-old man who is unable to close his mouth has an acute open lock deformity. This condition occurs when the condyle slips into a position anterior to the articular eminence and subsequently cannot return to the normal position. The most appropriate initial step in management of this patient is attempted manual reduction, which can be performed in the emergency department. Intravenous sedation is recommended to alleviate some of the muscle spasm. If reduction in the emergency department is unsuccessful, administration of succinylcholine in the operating room is the most appropriate next step, as reduction is almost always possible with patient sedation. If this problem persists, an MRI can be obtained to help in planning osseous reduction of the articular eminence. By reducing the slope of the eminence, the condyle returns to the articular fossa more easily, relieving the open lock deformity.

Intraarticular injection of a corticosteroid will not help to reduce the articular disk into its normal position and is administered only rarely in patients with inflammation resulting from an acute open lock deformity.

Disk plication involves plication or resection of the retrodiskal tissue to shorten the tissue and position the disk posteriorly over the condyle. It is not commonly used for treatment of open lock of the mandible.

Eminectomy is not an option in patients with acute open lock deformity of the mandible.

57
Q

Which of the following is the most common indication for performing distraction osteogenesis of the mandible in a 6-month-old infant?

(A) Malocclusion
(B) Mandibular hypoplasia with tongue-based airway obstruction
(C) Mandibular hypoplasia with tracheomalacia
(D) Unilateral craniofacial microsomia

A

The correct response is Option B.

In patients younger than age 2 years, mandibular distraction osteogenesis should only be performed when there is tongue-based airway compromise secondary to mandibular hypoplasia. In these patients, pulling the mandible forward will also pull the base of the tongue forward, relieving the airway obstruction.

Children younger than age 2 years with congenital hypoplasia or aplasia of select portions of the mandible but without airway compromise should not undergo distraction osteogenesis of the mandible because of the risk for permanent dental injury. In addition, mandibular procedures are associated with the potential for injury to the inferior alveolar nerve. Similarly, any procedures to correct malocclusion should only be performed after the permanent dentition has been established. Because children younger than age 1 year have either no dentition or rudimentary dentition and because the tooth buds are difficult to identify, operative correction of malocclusion is inappropriate.

Advancing the mandible and the base of the tongue will not relieve airway obstruction secondary to other causes, such as tracheomalacia or laryngomalacia. In neonates with these conditions, tracheotomy may be required for airway control.

Although distraction osteogenesis is typically performed in patients who have hemifacial microsomia (which can involve absence of the ramus, condyle, and/or glenoid fossa), it is only initiated in patients older than 1 year.

58
Q

In a patient undergoing LeFort I maxillary osteotomy and downfracture, what is the maximum bone defect (in mm) that will NOT require bone grafting?

(A) 1 mm
(B) 3 mm
(C) 5 mm
(D) 10 mm

A

The correct response is Option C.

Most surgeons believe that grafting should be performed for any bone defect of the craniofacial skeleton that is larger than 5 mm. In orthognathic surgery, any movement of the Le Fort I segment in excess of 5 mm without bone graft for support is likely to result in relapse.

59
Q

When obtaining lateral cephalograms, which of the following represents a commonly used cranial base plane?

(A) Long axis of the maxillary incisor plane
(B) Nasion-to-point A plane
(C) Occlusal plane
(D) Palatal plane
(E) Sella-nasion plane

A

The correct response is Option E.

A cephalogram is a radiograph obtained from a standard distance (typically 60 inches) with the head placed in a mechanical device. The lateral cephalogram is most effective for demonstrating both occlusion and relationships between the maxilla, mandible, and base of the skull.

A typical cephalometric plane connects three or more cephalometric points. The most commonly used cranial base planes are the sella-nasion, basion-nasion, and Frankfort horizontal planes. Commonly used maxillary planes include the long axis of the maxillary incisor plane; the nasion-to-point A plane; the occlusal plane, which extends between the mesial cusp of the maxillary molar through the point that bisects the overbite; and the palatal plane, which joins the anterior nasal spine to the posterior nasal spine.

60
Q

A 35-year-old woman is referred for evaluation of jaw pain. She reports a history of clicking and popping in her jaw particularly when chewing gum. She denies any history of trauma. Physical examination shows class I occlusion with a midline chin point. She has normal intra-oral opening. Which of the following diagnostic imaging techniques will provide the best sensitivity and specificity to evaluate her temporomandibular joint?

A) Arthrography
B) CT scan
C) Dynamic MRI
D) Fluoroscopy
E) Ultrasonography

A

The correct response is Option C.

This patient has subluxation of the disc causing her popping and subsequent pain. The study ordered must evaluate her for internal derangement of the disc related to the joint.

All of the listed imaging techniques have been used to evaluate temporomandibular joint (TMJ) disease. MRI is considered the gold standard for evaluation of the TMJ, particularly when evaluating the joint-disc relationship.

X-ray studies are indicated in the presence of trauma and would not provide the appropriate detail to determine the causes of this patient’s problems. CT scans are more sensitive and specific than conventional x-ray studies at determining bony abnormalities.

CT scan has a definite role in evaluation of patients with TMJ problems. CT scan should be enlisted when diagnosing bony abnormalities. It is useful in the diagnosis of ankylosis, osseous changes (e.g., idiopathic condylar resorption, or condylar hypoplasia seen in congenital anomalies such as Treacher Collins syndrome), or traumatic deformities. CT scan does not determine the location of the disc, so it would not be the appropriate imaging technique for this patient.

Arthrography is an invasive procedure and would not be performed for initial diagnosis. It can be combined with MRI to evaluate for adhesions or perforations.

Given its low cost and availability, ultrasonography of the TMJ has received increased attention in diagnosing internal derangement of the disc. Recent studies have reported a sensitivity of 65.8% and a specificity of 80.4% when compared with MRI. Ultrasonography was reported to be accurate in diagnosing normal disc position and the presence of abnormal disc-joint relationships but was less effective in evaluating disc displacement with or without reduction of the disc. Ultrasonography-guided arthrocentesis has been employed to decrease patient discomfort and multiple attempts to enter the joint space.

61
Q

An 18-year-old woman who underwent repair of cleft lip and palate during infancy is scheduled to undergo maxillary advancement surgery. Which of the following factors favors distraction osteogenesis over traditional orthognathic advancement?

(A) Anticipated advancement of 5 mm

(B) Anticipated advancement of 14 mm

(C) Need for simultaneous alveolar cleft bone grafting

(D) Need for simultaneous palatal expansion

A

The correct response is Option B.

Children with cleft lip often develop maxillary hypoplasia and Class III malocclusion. Traditional orthognathic advancement is limited by the tension of the soft-tissue envelope. Greater tension is associated with scarring, advancement distances of greater than 1 cm, and past pharyngeal surgical procedures. Any of these issues will limit the advancement and increase the relapse rate over the subsequent years.

Distraction osteogenesis allows the soft tissues to be expanded as the bone is advanced.

Palatal expansion requires horizontal movement. Distraction in multiple directions is difficult to control.

62
Q

A 25-year-old woman seeks surgical correction of a “gummy” smile. On examination, she has lip incompetence and full incisal show with the lips in repose and 3 mm of gingival show with animation. There is Angle class II malocclusion and a horizontal chin deficiency. These findings are most consistent with which of the following?

(A) Gingival alveolar hypertrophy
(B) Mandibular prognathism
(C) Mandibular retrognathism
(D) Vertical maxillary deficiency
(E) Vertical maxillary excess

A

The correct response is Option E.

This patient’s gummy smile is a manifestation of vertical maxillary excess. A gummy smile is defined as greater than 4 mm of incisal show with the lips in repose and greater than 2 mm of gingival show with animation. Patients with vertical maxillary excess, also known as the “long face” syndrome, have lip incompetence, an excessive interlabial gap, and mentalis muscle strain associated with attempts to overcome the labial incompetence. The chin is retruded and the mandible appears retrognathic secondary to a backward, or inferoposterior, high-angle clockwise mandibular rotation. The width of the inter-alar base is excessively narrow, and the nasolabial angle is open and obtuse. Appropriate management in this patient includes preoperative orthodontics followed by maxillary Le Fort I osteotomy and impaction, which will allow for autorotation and correction of the retrognathic appearance.

Gingival alveolar hypertrophy can result in excessive show of the teeth and gingiva but would not be responsible for all of the findings seen in this patient.

Mandibular prognathism is characterized by Angle class III malocclusion, effacement of the labiomental fold, and prominence of the lower third of the face. Several techniques may be considered depending on facial aesthetics, including mandibular setback, maxillary advancement, or a combination of both.

Patients with mandibular retrognathism have protrusion of the anterior teeth, associated lip incompetence, and a deep labiomental crease with eversion of the lower lip. There is Angle class II malocclusion; the lower third of the face may appear foreshortened. Mandibular advancement is recommended; genioplasty may also be required concomitantly.

Vertical maxillary deficiency manifests as a lack of maxillary incisor show, leading to an edentulous appearance. Affected patients have a concave facial profile with an “overclosed” appearance to the mandible. Other characteristics include excessive chin projection, an acute nasolabial angle, and an excessively wide inter-alar base. Appropriate management is Le Fort I osteotomy with interpositional grafting.

63
Q

A 17-year-old girl presents for correction of Angle class II malocclusion, 8 mm of overjet, and retrognathia/retrogenia. Mandibular advancement and genioplasty are planned. Which of the following is the most likely outcome of this procedure?

A) Decreased presence of the nasolabial folds
B) Decreased prominence of the labiomental crease
C) Increased area of the glossopharyngeal opening
D) Increased area of the velopharyngeal opening
E) Increased cervicomental angle

A

The correct response is Option C.

Understanding the anatomic changes after orthognathic surgery is crucial in helping to select the appropriate intervention as well as for informing patients about their expected results. Patients undergoing mandibular advancement (bilateral sagittal split osteotomy [BSSO]) and genioplasty surgery can expect predictable outcomes. The labiomental crease becomes deeper, while the cervicomental angle becomes more acute. Intraorally, the glossopharyngeal opening enlarges as the tongue is brought forward with the mandible. This can improve airway symptoms in patients with obstructive sleep apnea. Neither the nasolabial folds nor the velopharyngeal opening is affected with a mandibular advancement surgery.

64
Q

The Frankfort horizontal line passes through which of the following points?

(A) Gonion-pogonion
(B) Porion (tragion)-nasion
(C) Porion (tragion)-orbitale
(D) Sella-nasion
(E) SNA-SNB

A

The correct response is Option C.

The Frankfort horizontal line passes through the porion (tragion) and orbitale. Anatomists in Germany in the last century determined this point to be a horizontal reference line for skull orientation. In addition, the sella-nasion line is used as a reference line; it is oriented at 6 to 8 degrees from the Frankfort horizontal. This reference line is used to define the length of the cranial base.

Maxillary relations can be evaluated using the anterior and posterior nasal spines, which can be used for maxillary orientation, as well as the SNA angle. This measures the position of point A (anterior maxilla) relative to the anterior cranial base (SN). A normal SNA angle is identified as 82 degrees  4 degrees. Decreased width indicates maxillary retrusion, while increased width indicates maxillary protrusion.

In contrast, the gonion-pogonion, which represents the mandibular plane, and the SNB angle can be used to evaluate mandibular relations. The SNB angle measures the position of point B (anterior mandible) relative to the anterior cranial base (SN). A normal angle is defined as 79 degrees  3 degrees. A wide angle denotes mandibular protrusion, while a narrow angle denotes inadequate mandibular development.

65
Q

A 17-year-old boy is referred for orthognathic surgery. On intraoral examination, the mesiobuccal cusp of the maxillary first molar is positioned distal to the buccal groove of the mandibular first molar. Which of the following best describes this occlusal relationship?

(A) Angle class I
(B) Angle class II
(C) Angle class III
(D) Overbite
(E) Overjet

A

The correct response is Option C.

Occlusion describes the relationship between the upper and lower teeth and is critical for planning reconstructive surgery of the jaws. The Angle classification of occlusion is based on the relationship of the mesiobuccal cusp of the maxillary first molar to the mandibular first molar when viewed in the sagittal plane. In this patient, the mesiobuccal cusp of the maxillary first molar lies distal (posterior) to the buccal groove of the mandibular first molar and instead is located in the buccal groove of the lower second molar, which is Angle class III malocclusion.

In patients with class I (normal) occlusion, the mesiobuccal cusp of the maxillary first molar lies in the buccal groove of the mandibular first molar. Angle class II malocclusion is defined as the mesiobuccal cusp of the maxillary first molar located mesial (anterior) to the buccal groove of the mandibular first molar. This classification of malocclusion has two divisions; in class II, division 1, the lateral incisors are flared labially, while in class II, division 2, the incisors are lingually inclined.

Overbite is a vertical measurement referring to the distance between the maxillary incisor edge and the mandibular incisor edge with the teeth in centric occlusion. Overjet is a horizontal measurement referring to the distance between the incisal aspect of the maxillary incisors and the incisal aspect of the mandibular incisors with the teeth in centric occlusion.

66
Q

A 24-year-old woman is referred to the office by her orthodontist for evaluation of facial disharmony. The following angles are obtained on cephalometric analysis:
SNA 70 (normal = 81.2)
SNB 77 (normal = 77.3)
SN-pogonion 87 (normal = 80)
Which of the following procedures is most effective to achieve facial symmetry in this patient?
(A) Le Fort I advancement and advancement genioplasty
(B) Le Fort I advancement and setback genioplasty
(C) Sagittal split mandibular advancement
(D) Sagittal split mandibular setback
(E) Vertical ramus osteotomy and setback

A

The correct response is Option B.

The cephalogram is a standardized radiograph used for analysis of facial disharmony and asymmetry. Labeled landmarks help establish the relationship of the maxilla, mandible, and skull base to other facial structures. The sella (S) point marks the center of the hypophyseal fossa. The nasion (N) is the junction of the nasal and frontal bones at the most posterior point of the curve of the nose. The A point marks the innermost curvature from the maxillary anterior nasal spine to the alveolar process. The angle created by these points (SNA) establishes the maxillary position in relation to the skull base. The B point is located at the innermost curvature from the chin to the alveolar process of the mandible. The SNB angle similarly establishes the mandibular relationship to the skull base.

The prominence of the chin is often an important consideration in orthognathic surgery. The pogonion (Pg) is the most anterior chin point. The SNPg angle is representative of the degree of chin prominence relative to the SNB (mandible position).

Normal angle values are given in the text. This patient has a relatively retrusive maxilla and a normally positioned mandible. This would represent a class III relation. The pogonion is anteriorly displaced in relation to both the mandible and the maxilla.

Therefore, to establish better facial relationships, the maxilla should be advanced at the Le Fort I level and the chin setback via a genioplasty. The mandible does not need to be moved.

67
Q

A 23-year-old woman seeks improvement in the appearance of her smile. A Le Fort I osteotomy is planned for correction of maxillary excess. Which of the following arteries is at increased risk for injury during this procedure?

A) Ascending pharyngeal artery
B) Descending palatine artery
C) Infraorbital artery
D) Posterior superior alveolar artery
E) Sphenopalatine artery

A

The correct response is Option B.

The blood supply to the maxilla before Le Fort I osteotomy is from the descending palatine, infraorbital, and posterior superior alveolar arteries, which are branches of the internal maxillary artery off the external carotid artery. When performing a Le Fort I osteotomy, the descending palatine artery runs vertically in the posterior maxilla and is at risk of injury.

After Le Fort I osteotomy, the blood supply to the maxilla is from the ascending palatine artery, which is a branch of the facial artery, off the external carotid artery and the palatine branch of the ascending pharyngeal artery, off the external carotid artery.

68
Q

A 13-year-old boy who underwent repair of left unilateral cleft lip and palate is brought to the office because he is dissatisfied with his “underbite.” He reports no other symptoms. Cephalometric analysis shows an SNA angle of 76 degrees (N = 81 ± 3) and an SNB angle of 81 degrees (N = 79 ± 3). A negative overjet of 5 mm is noted. Which of the following is the most appropriate management?

A ) Bimaxillary advancement
B ) Le Fort I advancement of the maxilla with internal fixation
C ) Le Fort I osteotomy and distraction
D ) Mandibular setback
E ) Orthodontics and follow-up in 1 year

A

The correct response is Option E.

At 13 years of age, the facial skeletal growth is not complete. Therefore, the patient should wait until his facial skeletal growth is complete, usually at age 18 for boys. Meanwhile, he should follow up regularly with his craniofacial team. The Angle class III malocclusion is not uncommonly seen in patients after cleft palate repair. The scarring from bony dissection in the palate repair restricts maxillary growth. However, unless there are severe symptoms (such as respiratory compromise), the definitive orthognathic surgery is deferred until skeletal maturation. When skeletal maturation is complete, a Le Fort I osteotomy with immediate fixation, or distraction osteogenesis (DO), can be done. DO is reserved for those cases where the advancement is calculated to exceed 10 mm. His SNB angle is within normal limits, therefore a setback is not indicated, nor is bimaxillary surgery.

69
Q

A 29-year-old woman comes to the office for evaluation of orthognathic profile and class I occlusion. Physical examination shows isolated retrogenia and moderate vertical mandibular excess. Which of the following types of genioplasty is most appropriate?
(A) Advancement with horizontal osteotomy only
(B) Advancement with horizontal osteotomy and downgrafting
(C) Advancement with inferiorly angled osteotomy
(D) Alloplastic augmentation, extraoral approach
(E) Alloplastic augmentation, intraoral approach

A

The correct response is Option C.

The most appropriate genial treatment option is advancement with the osteotomy angled inferiorly. The angle of the osteotomy has an impact on the vertical dimension of the mandible as the segment is advanced forward. An osteotomy angled inferiorly in relation to the occlusal plane will provide a progressive decrease in the vertical dimension as the osteotomy segment is advanced. A 2- to 4-mm reduction in chin height can be achieved with this technique. When a larger height reduction (in excess of 5 mm) is indicated, a horizontal wafer of bone is removed above the horizontal sliding osteotomy. A horizontal osteotomy, relatively parallel to the occlusal plane, would provide a more pure anteroposterior movement.

Downgrafting the osteotomy site, with an interpositional bone graft or alloplastic material, would be indicated to increase the vertical dimension in a patient with vertical mandibular deficiency.

Alloplastic augmentation alone, whether placed through an intraoral or extraoral approach, is best indicated for a unidimensional change, such as a pure sagittal deficiency.

70
Q

Which of the following cephalometric planes extends from the most superior aspect of the external auditory canal through the inferior orbital rim?

A) Facial plane angle
B) Frankfort horizontal plane
C) Nasion-pogonion plane
D) Sella-nasion plane

A

The correct response is Option B.

Numerous points, angles, and planes may be identified on a standard cephalogram. The sella is the midpoint within the sella turcica. The nasion is the most anterior point of the frontonasal suture in the midsagittal plane. The pogonion is the most forward-projecting point on the anterior surface of the chin. The Frankfort horizontal is a line extending from the porion (the superior extent of the ear canal) to the orbitale (inferior extent of the orbit). The Frankfort horizontal was originally introduced at an anthropological conference in Frankfurt, Germany, in 1884. It is used to orient the craniofacial skeleton and serve as a relation to other descriptive planes. The facial plane angle extends from the nasion to the pogonion. It is used to measure the degree of protrusion or retrusion of the lower jaw.

71
Q

A 44-year-old man is referred by his pulmonologist for consultation regarding surgical management of obstructive sleep apnea. He is unable to tolerate treatment with continuous positive airway pressure. History includes septoplasty to correct a deviated nasal septum and uvulopalatopharyngoplasty 2 years ago. History also includes orthodontic treatment 30 years ago. Current examination shows a Class I molar relationship with normal overjet. Cephalometric analysis shows an SNA angle of 76.7 degrees (N 80), an SNB angle of 72.1 degrees (N 78), and a posterior airway space (PAS) of 4 mm (N 11.0 ± 1). Orthognathic surgery is planned. Which of the following is the most appropriate management?

A) Mandibular advancement
B) Maxillary advancement
C) Maxillary and mandibular advancement
D) Maxillary/palatal expansion

A

The correct response is Option C.

Options for management of obstructive sleep apnea (OSA) include medical and surgical modalities. The first line of treatment is continuous positive airway pressure. Surgical treatments are directed toward the level of pathology(ies); ie, nose, palate, or base of the tongue. The goal of orthognathic surgery in the treatment of OSA is the enlargement and decreased collapse of the velo-oropharyngeal airway through anterior displacement of the soft tissues and musculature. Orthognathic treatment in adult OSA patients generally involves maxillary and mandibular advancement. The amount of advancement is usually greater than 10 mm regardless of cephalometric analysis. Success rates seen with simultaneous large advancement of the maxilla and mandible are between 75 and 100%.

72
Q

A 23-year-old woman undergoes Le Fort I osteotomy with impaction of the maxilla because of vertical maxillary excess. The final vertical position of the maxilla is most accurately determined using which of the following studies?

(A) Assessment of maxillary lip-tooth relationship
(B) Cephalometric analysis of the ANB angle
(C) Cephalometric analysis of the SNA angle
(D) Dental model surgery
(E) Panorex radiography

A

The correct response is Option A.

Assessment of the maxillary lip-tooth relationship is the most accurate study in determining the final vertical position of the maxilla. This assessment can be made on a cephalometric radiograph by performing a soft-tissue and skeletal analysis and prediction tracing. Intraoperative assessment of the amount of tooth show is also important in determining the final vertical position of the maxilla. Normally, the lip-tooth relationship is 2 to 3 mm of tooth show.

Cephalometric analysis of the ANB and SNA angles are skeletal measurements. The ANB angle relates the maxilla to the mandible in the horizontal plane. The SNA angle relates the maxilla to the base of the cranium in the horizontal plane. Neither measurement assesses the vertical position of the maxilla or the soft-tissue envelope.

73
Q

During unilateral distraction osteogenesis of the mandible in a 5-year-old child, which of the following is the most appropriate technique to optimize the resulting bone formation?

(A) Acute intraoperative distraction of 5 mm
(B) Nonrigid pin fixation of the bone segments
(C) Resection of periosseous soft tissues
(D) Low-energy corticotomy
(E) Wide subperiosteal dissection

A

The correct response is Option D.

In experimental studies of limb lengthening involving the long bones, minimal disruption of the central medullary bone has been shown to be a core principle of distraction osteogenesis. This can be accomplished using a low-energy corticotomy that divides only the bone cortex, thus optimizing the resultant bone formation.

Acute intraoperative distraction of 5 mm or more is likely to result in early disruption of the medullary bone and impairment of the central callus. Fibrous union has been demonstrated following acute intraoperative distraction in animal models.

Because increased fixator stability has been shown to improve the quality of regenerated bone, nonrigid pin fixation of bone segments is discouraged because it would impair resultant bone formation.

Enhanced bone formation has been shown to result from preservation of the periosseous and intraosseous soft tissues. Several authors have advocated against resecting soft tissue because the vascularity of the periosseous soft tissue is thought to be critical to bone regeneration.

Wide subperiosteal undermining is also thought to be detrimental to bone regeneration.

74
Q

Le Fort I osteotomy is performed through which of the following structures?

(A) Lateral orbital wall
(B) Maxillary sinus
(C) Medial orbital walls
(D) Nasofrontal junction
(E) Orbital floor

A

The correct response is Option B.

The Le Fort classification is used to identify the pattern of midface fractures. Identification of the lines of fracture is useful in planning osteotomies for patients requiring midface advancement.

Le Fort I midface advancement involves placement of the osteotomy at a level above the apices of the teeth. The entire alveolar processes of the maxilla, vault of the palate, and pterygoid processes are included in a single block. The osteotomy extends transversely across the base of the maxillary sinus and the floor of the piriform aperture.

The Le Fort II osteotomy begins above the level of the apices of the teeth laterally and extends through the pterygoid plates in a manner similar to the Le Fort I osteotomy, leaving a central maxillary segment undisturbed. The osteotomy procedure includes portions of the medial orbital walls, orbital floor, and nasofrontal junction. Patients undergoing Le Fort II midface advancement are at increased risk for injury to the ethmoid area and the lacrimal system. The lateral orbital wall is unaffected.

Le Fort III osteotomy extends through the zygomaticofrontal suture and the nasofrontal suture and across the orbital floor. The entire midface is completely detached from the base of the skull.

75
Q

Which of the following tooth root apexes is at greatest risk for damage during a Le Fort I osteotomy for maxillary advancement?

A ) Bicuspid

B ) Canine

C ) Central incisor

D ) First molar

E ) Lateral incisor

A

The correct response is Option B.

The canine tooth root is the longest root extending onto the maxillary wall near the piriform rim. The length of the canine tooth from the incisal edge to the root apex is approximately 30 mm. The root is at risk during both osteotomy and plating and serves as an important landmark in operative planning. The roots of central and lateral incisors and bicuspids and molars are at less risk because they have shorter roots. Crowns of the central and lateral incisors are at greater risk during disimpaction of the Le Fort I segment if the maxillary disimpaction forceps are not placed properly. With careful osteotomy planning, disimpaction and plating dental injury are infrequent occurrences.

76
Q

A 17-year-old girl with Marfan syndrome comes to the office for an orthognathic evaluation. Intraoral examination shows a bilateral posterior lingual crossbite. Which of the following is the most appropriate management?
A) Le Fort I osteotomy with palatal expansion
B) Le Fort I osteotomy with palatal narrowing
C) Mandibular osteotomy with narrowing
D) Mandibular osteotomy with widening

A

Correct answer A.

Patients with Marfan syndrome typically have a high-arched and narrow palate resulting in a transversely constricted maxilla. Le Fort I osteotomy with palatal expansion is an appropriate surgical option for correcting the skeletal facial disharmony in the patient described.

77
Q

A 16-year-old girl with a history of rheumatoid arthritis has mandibular retrusion and an anterior open bite. Skeletal maturity is complete. Which of the following is most appropriate for correction of the deformity?

(A) Use of orthodontic functional appliances
(B) Advancement sagittal split osteotomy and advancement genioplasty
(C) Maxillary impaction and advancement genioplasty
(D) Maxillary impaction, advancement sagittal split osteotomy, and genioplasty

A

The correct response is Option C.

Because episodes of juvenile rheumatoid arthritis can be triggered by increased loading on the temporomandibular joint, orthognathic procedures should be limited to those that minimize condylar load. Therefore, the most appropriate procedure for correction of this child’s deformity is maxillary impaction, which will allow closure of the open bite without creating condylar load. The resultant mandibular autorotation of the mandible will increase the SNB (sella-nasion-point B) angle, thereby correcting the mandibular deficiency. Any subsequent sagittal deficiency of the mandible can be improved with advancement genioplasty.

Orthodontic functional appliances are not effective in a 16-year-old patient and are relatively contraindicated in patients with juvenile rheumatoid arthritis because of the forces exerted on the temporomandibular joint. Mandibular advancement will result in increased condylar load, leading to degenerative remodeling.

Advancement sagittal split osteotomy is less acceptable than maxillary impaction and advancement genioplasty because it will increase condylar load.

78
Q

A 34-year-old woman undergoes a down-fracture of the maxilla during Le Fort I osteotomy. Profuse bleeding is noted in the posterior aspect of the lateral nasal wall. Injury to which of the following arteries is most likely?
(A) Anterior ethmoidal
(B) Descending palatine
(C) Greater palatine
(D) Infraorbital
(E) Nasopalatine

A

The correct response is Option B.

The descending palatine artery is a branch of the third portion of the internal maxillary artery. It descends vertically within the perpendicular portion of the palatine bone. Injury to this vessel is not uncommon while performing a Le Fort I osteotomy. Division of this artery has been shown not to expose the maxilla to necrosis.

79
Q

A 15-year-old girl who has midface hypoplasia. Which of the following best describes this patient’s occlusion? **no photo given**

(A) Angle class I
(B) Angle class II, division 1
(C) Angle class II, division 2
(D) Angle class III
A

The correct response is Option D.

The Angle classification of occlusion is based on the relationship of the mesiobuccal cusp of the maxillary first molar to the mandibular first molar when viewed in the sagittal plane. Malocclusions can occur as a result of dental dysplasia, skeletal dysplasia, or a combination of both.
The findings shown in the photograph are consistent with Angle class III malocclusion, or mesio-occlusion. In this patient, the mesiobuccal cusp of the maxillary first molar lies distal or posterior to the buccal groove of the mandibular first molar and in the buccal groove of the lower second molar, resulting in the prognathic profile. With this type of malocclusion, the maxillary teeth are typically positioned end-to-end or in a cross-bite pattern. Angle class III malocclusion is also common in patients with cleft palate.

In patients with class I occlusion (neutral occlusion), the mesiobuccal cusp of the maxillary first molar lies in the buccal groove of the mandibular first molar. There is a minimal degree of overjet and overbite.

Angle class II malocclusion, or distal occlusion, is defined as the mesiobuccal cusp of the maxillary first molar being located anterior to the buccal groove of the mandibular first molar. This classification of malocclusion has two divisions; in class II, division 1, the lateral incisors are flared labially, resulting in significant overjet, while in class II, division 2, the incisors are lingually inclined, producing a retrognathic appearance.

80
Q

A 24-year-old woman comes to the office for consultation regarding her appearance when smiling. Physical examination shows the mesiobuccal cusp of the first maxillary molar lying distal to the buccal groove of the first mandibular molar. Which of the following is the most appropriate Angle classification?
(A) I
(B) IIA
(C) IIB
(D) III

A

The correct response is Option D.

Occlusion as described by the Angle classification uses the relationship of the permanent first molars of the maxilla and mandible as its reference point. Patients with class I occlusion (normal) have the mesiobuccal cusp of the maxillary first molar lying in the buccal groove of the mandibular first molar. Patients with class II occlusion have the mesiobuccal cusp of the maxillary first molar located mesial to the buccal groove of the mandibular first molar. Class III occlusion, as illustrated in this case, is described as having the mesiobuccal cusp of the maxillary first molar positioned distal to the buccal groove of the mandibular first molar.

81
Q

An 18-year-old man who was born with a bilateral cleft lip and palate, which were repaired in infancy, has completed orthodontic care and treatment. Maxillary advancement is planned. Which of the following is the most likely dental relationship in this patient?

(A) Negative overbite

(B) Positive overbite

(C) Negative overjet

(D) Positive overjet

A

The correct response is Option C.

Both overbite and overjet describe the relationship of the maxillary incisors to the mandibular incisors. Cephalometric angles are frequently used to analyze the relationship of the teeth to the maxilla and mandible.

Overbite is a vertical measure of the overlapping of the upper incisors to the lower incisors. It is calculated by measuring in millimeters the vertical relationship of the perpendicular plane to the occlusal plane. This value is positive when overlap is present and negative when there is no overlap (as in the anterior open bite of Apert syndrome).

Overjet is a horizontal measure of the position of the upper and lower incisors in the anteroposterior plane. Overjet is also measured in millimeters from the labial portion of the incisal edge of the upper incisor to the labial edge of the lower incisor. This number is positive if the maxillary teeth are forward of the mandibular teeth and negative if the lower teeth are forward of the upper teeth.

For the patient with a Class III dental relationship, in which the maxilla is retrusive, overbite is unaffected and the overjet is most likely negative.

82
Q

An 18-year-old man undergoes Le Fort I advancement for correction of a 10-mm maxillomandibular discrepancy as a result of maxillary hypoplasia. Which of the following diagnoses places this patient at greatest risk for postoperative development of velopharyngeal incompetence?

A ) Apert syndrome

B ) Clefting of the lip and palate

C ) Craniofacial microsomia

D ) Mandibular prognathism

A

The correct response is Option B.

Substantial advancement of the maxilla in patients with mid face hypoplasia secondary to a repaired cleft of the lip and palate is a risk factor for the development of velopharyngeal incompetence. Key preoperative findings to evaluate the risk include nasal air emission, nasal resonance, borderline velopharyngeal incompetence, or a combination of findings. In patients with an Angle class III malocclusion due to mandibular prognathism, mandibular setback, rather than maxillary advancement, is the procedure of choice. This should not increase a patient €™s risk for development of velopharyngeal incompetence. Patients undergoing maxillary advancement for other craniofacial anomalies, such as craniofacial microsomia or Apert syndrome, are at lower risk for development of velopharyngeal incompetence.

83
Q

Which of the following percentages best represents the incidence of paresthesia of the lower lip immediately after bilateral sagittal split osteotomy?
(A) 10%
(B) 30%
(C) 50%
(D) 70%
(E) 90%

A

The correct response is Option E.

Paresthesia of the lower lip is the most common immediate postoperative finding following a bilateral sagittal split osteotomy. It is generally bilateral and is due to neurapraxia resulting from stretch and compression of the inferior alveolar nerve as the mandible is mobilized and fixed into its new position. Studies have shown that the incidence of this finding ranges from 85% to 97% in the immediate postoperative period. In one study, 55% of the patients reported some degree of paresthesia at one month, which was further reduced to 12.5% at one year. The older the patient, the more protracted the sensory deficit.

84
Q

An 18-year-old man is evaluated because of an overbite. Cephalometric analysis shows an SNA angle of 83 degrees (N 82 ± 3) and an SNB angle of 74 degrees (N 80 ± 3). Which of the following is the most likely underlying cause of this condition?

A ) Prognathic maxilla

B ) Retrognathic maxilla

C ) Prognathic mandible

D ) Retrognathic mandible

A

The correct response is Option D.

An €œoverbite € (Angle class II malocclusion) may be caused by several different etiologies: a prognathic maxilla, a retrognathic mandible, or both; even a prognathic mandible with a more severely prognathic maxilla, or a retrognathic maxilla with a more severely retrognathic mandible, is possible. The patient described is exhibiting isolated mandibular deficiency, or retrognathia, which is characterized by a decreased sella-nasion-point B (SNB) angle combined with a normal sella-nasion-point A (SNA) angle. The SNA and SNB angles determine the position of the maxilla and mandible relative to the cranial base. The SNA angle measures the position of point A (subspinale) relative to the anterior cranial base with the normal value being 82 degrees plus or minus 3 degrees. The SNA angle is increased in maxillary prognathism and decreased in maxillary retrognathism. The SNB angle measures the position of point B (supramentale) relative to the anterior cranial base with the normal value being 80 degrees plus or minus 3 degrees. The SNB angle is increased in mandibular prognathism and decreased in maxillary retrognathism.

85
Q

Which of the following is the best method to treat maxillary transverse deficiency in a skeletally mature patient?

A) Mandibular setback (bilateral sagittal split osteotomy)
B) Maxillary advancement (Le Fort I advancement)
C) Orthopedic and orthodontic expansion
D) Reverse-pull headgear
E) Surgically assisted rapid palatal expansion

A

The correct response is Option E.

Maxillary transverse deficiency (MTD) in the skeletally mature patient is best addressed with surgically assisted rapid palatal expansion (SARPE). In the young patient (before suture closure), orthopedic and orthodontic forces can be more easily used to correct the MTD.

Reverse-pull headgear does not aid in expansion in the skeletally mature patient.

One-piece Le Fort and bilateral sagittal split osteotomy (BSSO) procedures address anterior-posterior discrepancies rather than transverse deficiencies.

86
Q
A 24-year-old woman is referred to the office by her orthodontist for evaluation of facial disharmony. The following angles are obtained on cephalometric analysis:
SNA 70 (normal = 81.2)
SNB 77 (normal = 77.3)
SN-pogonion 87 (normal = 80) 
Which of the following procedures is most effective to achieve facial symmetry in this patient?

(A) Le Fort I advancement and advancement genioplasty
(B) Le Fort I advancement and setback genioplasty
(C) Sagittal split mandibular advancement
(D) Sagittal split mandibular setback
(E) Vertical ramus osteotomy and setback

A

The correct response is Option B.

The cephalogram is a standardized radiograph used for analysis of facial disharmony and asymmetry. Labeled landmarks help establish the relationship of the maxilla, mandible, and skull base to other facial structures. The sella (S) point marks the center of the hypophyseal fossa. The nasion (N) is the junction of the nasal and frontal bones at the most posterior point of the curve of the nose. The A point marks the innermost curvature from the maxillary anterior nasal spine to the alveolar process. The angle created by these points (SNA) establishes the maxillary position in relation to the skull base. The B point is located at the innermost curvature from the chin to the alveolar process of the mandible. The SNB angle similarly establishes the mandibular relationship to the skull base.

The prominence of the chin is often an important consideration in orthognathic surgery. The pogonion (Pg) is the most anterior chin point. The SNPg angle is representative of the degree of chin prominence relative to the SNB (mandible position).

Normal angle values are given in the text. This patient has a relatively retrusive maxilla and a normally positioned mandible. This would represent a class III relation. The pogonion is anteriorly displaced in relation to both the mandible and the maxilla.

Therefore, to establish better facial relationships, the maxilla should be advanced at the Le Fort I level and the chin setback via a genioplasty. The mandible does not need to be moved.

87
Q

A 34-year-old woman undergoes a down-fracture of the maxilla during Le Fort I osteotomy. Profuse bleeding is noted in the posterior aspect of the lateral nasal wall. Injury to which of the following arteries is most likely?
(A) Anterior ethmoidal
(B) Descending palatine
(C) Greater palatine
(D) Infraorbital
(E) Nasopalatine

A

The correct response is Option B.

The descending palatine artery is a branch of the third portion of the internal maxillary artery. It descends vertically within the perpendicular portion of the palatine bone. Injury to this vessel is not uncommon while performing a Le Fort I osteotomy. Division of this artery has been shown not to expose the maxilla to necrosis.

88
Q

Which of the following soft-tissue changes is most common in patients who have undergone Le Fort I maxillary advancement?

(A) Decreased incisor show
(B) Increased nasolabial angle
(C) Lengthening of the upper lip
(D) Narrowing of the alar base

A

The correct response is Option B.

In patients who have undergone Le Fort I maxillary advancement, characteristic soft-tissue changes in the nose include an increased nasolabial angle and widened alar base. In addition, the upper lip is shortened and incisal show is increased. These changes vary depending on the degree of maxillary advancement and the proclination of the maxillary incisors. Alar cinch sutures can be used to minimize excess widening of the alar base, while excess shortening of the upper lip may be prevented by performing V-Y advancement during closure.

89
Q

Which of the following is the most appropriate initial step in the management of a patient being evaluated for chin implantation?

(A) Cephalometric analysis
(B) Determination of the patient’s occlusion
(C) Evaluation of bite mechanics
(D) Panoramic radiograph

A

The correct response is Option B.

Dental occlusion must be adequately assessed in any patient who is being considered for chin implantation. If occlusion is normal, orthognathic surgery is not necessary. However, if Angle class III malocclusion is the underlying cause of the retrognathia, chin implantation is not appropriate because it does not address the underlying malformation.

If the patient is shown to have malocclusion, cephalometric analysis, evaluation of bite mechanics, and a panoramic radiograph may be required.

90
Q

A 16-year-old girl with facial asymmetry secondary to hemifacial microsomia comes to the office for evaluation of orthognathic surgery. Which of the following procedures puts her at the highest risk for perioperative bleeding?

A) Bilateral sagittal split osteotomy
B) Distraction osteogenesis
C) Le Fort I osteotomy
D) Mandibular vertical ramus osteotomy
E) Osseous genioplasty

A

The correct response is Option C.

Significant hemorrhage is uncommon in orthognathic surgery, but when it occurs, it is most likely secondary to the maxillary osteotomies. The vessels at risk with the maxillary osteotomy include the greater palatine vessels, maxillary artery, and pterygoid plexus. The incidence of significant hemorrhage with mandible osteotomies is rare. The vessels at risk include the inferior alveolar artery, facial artery, retromandibular vein, and the pterygoid venous vein. Distraction osteogenesis is associated with lower risk for bleeding than any of the open procedures.

91
Q

Which of the following findings is most common in patients with vertical maxillary excess?

A) Counterclockwise rotation of the mandible
B) Excessive height in the upper half of the face
C) Mentalis strain
D) Posterior open bite
E) Retrusive midface

A

The correct response is Option C.

Vertical maxillary excess (VME), or long face syndrome, occurs when there is excessive (imbalanced) anterior facial height in the lower half of the face. The midface is relatively protrusive. Excessive eruption of the posterior dentition in the maxilla can cause clockwise rotation of the mandible. There is lip incompetence, excessive gingival show, and an effort to close the lips can result in mentalis strain. It is associated with an anterior open bite.

92
Q

A 20-year-old woman is undergoing evaluation because she has a gummy smile. On examination, she has a long, narrow nose with an obtuse nasolabial angle. There is lip incompetence, excessive show of the upper incisors with the lips in repose, and mentalis muscle strain with lip closure. She has Angle class II malocclusion. Cephalometric analysis shows a normal SNB angle.

Which of the following is the most likely diagnosis?

(A) Mandibular deficiency
(B) Mandibular excess
(C) Vertical maxillary deficiency
(D) Vertical maxillary excess

A

The correct response is Option D.

The findings in this patient are consistent with vertical maxillary excess, or long-face syndrome. Patients with vertical maxillary excess have increased height of the lower third of the face. This condition is characterized by labial incompetence with the lips in repose, excessive incisal show, and a lip-to-tooth relationship greater than 3 mm. Mentalis muscle strain occurs as the patient attempts to obtain labial competence. In addition, the nose and alar bases are narrowed and the nasolabial angle is obtuse. The chin is retruded and vertically long. The mandible is retrognathic, which may occur secondary to true retrognathia or backward autorotation. Occlusion is Angle class II.

Mandibular deficiency is associated with true retrognathia. Although the physical findings, such as Angle class II malocclusion and lip incompetence, are similar to vertical maxillary excess, the SNB angle is decreased in patients with mandibular deficiency.

Mandibular excess, or prognathism, is characterized by a wide lower third of the face and a full, prominent lower lip below the vermillion border. Intraoral examination shows an anterior crossbite and Angle class III malocclusion. Excessive closure of the jaws results in decreased vertical facial height. Cephalometric analysis shows an increased SNB angle.

Patients with vertical maxillary deficiency, or short-face syndrome, have decreased facial height vertically, absence of maxillary incisor show with an edentulous look, an upper lip that appears short and flat, a deep bite with an excessively protruding chin, wide alar bases, and an acute mandibular plane angle. This is Angle class III malocclusion. The SNB angle is normal or larger than normal on cephalometric analysis.

93
Q

A 29-year-old woman comes to the office for evaluation of orthognathic profile and class I occlusion. Physical examination shows isolated retrogenia and moderate vertical mandibular excess. Which of the following types of genioplasty is most appropriate?

(A) Advancement with horizontal osteotomy only
(B) Advancement with horizontal osteotomy and downgrafting
(C) Advancement with inferiorly angled osteotomy
(D) Alloplastic augmentation, extraoral approach
(E) Alloplastic augmentation, intraoral approach

A

The correct response is Option C.

The most appropriate genial treatment option is advancement with the osteotomy angled inferiorly. The angle of the osteotomy has an impact on the vertical dimension of the mandible as the segment is advanced forward. An osteotomy angled inferiorly in relation to the occlusal plane will provide a progressive decrease in the vertical dimension as the osteotomy segment is advanced. A 2- to 4-mm reduction in chin height can be achieved with this technique. When a larger height reduction (in excess of 5 mm) is indicated, a horizontal wafer of bone is removed above the horizontal sliding osteotomy. A horizontal osteotomy, relatively parallel to the occlusal plane, would provide a more pure anteroposterior movement.

Downgrafting the osteotomy site, with an interpositional bone graft or alloplastic material, would be indicated to increase the vertical dimension in a patient with vertical mandibular deficiency.

Alloplastic augmentation alone, whether placed through an intraoral or extraoral approach, is best indicated for a unidimensional change, such as a pure sagittal deficiency.

94
Q

In patients with vertical maxillary excess undergoing Le Fort osteotomy with maxillary impaction, which of the following findings is most likely postoperatively?

(A) Increased mentalis strain
(B) Increased upper incisal show
(C) More obtuse nasolabial angle
(D) Retrogenia
(E) Widened alar base

A

The correct response is Option E.

Patients with vertical maxillary excess, or long-face syndrome, have a narrow alar base, an obtuse nasolabial angle, and an anterior open bite. Mentalis muscle strain and labial incompetence are increased, and there is excess gingival show and exposure of the upper incisors.

Appropriate management is Le Fort I osteotomy with maxillary impaction; osseous genioplasty is also performed in some patients. These procedures will correct many of the findings associated with this condition, including decreasing the mentalis muscle strain and incisal show and creating a more acute nasolabial angle. The alar base will be widened. Le Fort I osteotomy also rotates the mandible forward and upward, resolving the retrogenia associated with long-face syndrome. Postoperative lateral cephalograms will show forward autorotation of the mandible with counterclockwise rotation.

95
Q

A 27-year-old woman desires surgical correction because she has an edentulous appearance. Physical examination shows decreased height in the lower third of the face and absence of maxillary incisor show. Cephalometric analysis shows an acute mandibular plane angle.

Which of the following operative procedures is most appropriate?

(A) Genioplasty
(B) Le Fort I osteotomy with inferior repositioning
(C) Le Fort I osteotomy with maxillary impaction
(D) Le Fort III osteotomy
(E) Bilateral sagittal split osteotomy of the mandible

A

The correct response is Option B.

This patient has findings consistent with vertical maxillary deficiency, or short face syndrome. Affected patients have a vertical decrease in facial height and absence of maxillary show, resulting in an edentulous appearance. The upper lip appears short and flat; the bite is deep, and the chin protrudes excessively. The alar bases are wide. The mandibular plane angle is acute.

Appropriate management of vertical maxillary deficiency is Le Fort I osteotomy with inferior repositioning of the maxilla. Bone grafts can be interposed to stabilize the mandible during downward movements and prevent recurrence of the deformity.

Genioplasty can be performed as an adjuvant to Le Fort I osteotomy to improve the aesthetic appearance of the chin but will not effectively increase maxillary height if performed alone.

Le Fort I osteotomy with maxillary impaction is performed to shorten the face in patients with vertical maxillary excess.

Le Fort III osteotomy is not indicated for patients with deformities limited to the lower third of the face.

Bilateral sagittal split osteotomy can be combined with downfracture of the maxilla to improve facial projection but will not improve midface height when performed alone.

96
Q

A 25-year-old woman seeks surgical correction of a “gummy” smile. On physical examination, she has lip incompetence, 5 mm of incisor show at rest, evidence of mentalis muscle strain, and Angle class II malocclusion. She has a long, narrow nose, a retruded chin, and excessive vertical height in the lower third of the face.

Which of the following is the most appropriate surgical correction?

(A) Anterior segmental maxillary osteotomy with impaction
(B) Le Fort I osteotomy with inferior repositioning and genioplasty
(C) Le Fort I osteotomy with maxillary impaction and genioplasty
(D) Le Fort III osteotomy
(E) Sagittal split osteotomy and genioplasty

A

The correct response is Option C.

This patient has findings consistent with vertical maxillary excess, including increased height in the lower third of the face and a narrow nose. Patients with this condition frequently have constriction of the alar base and an obtuse nasolabial angle. There is lip incompetence and incisor show at rest; the lip-to-tooth ratio is greater than 3 mm. Mentalis muscle strain occurs as the patient attempts to obtain lip competence. This chin is retruded and vertically long; occlusion is frequently Angle class II. The mandible is typically retrognathic; this may be true retrognathism or may result from backward autorotation of the jaw.

Following application of orthodontic appliances to eliminate any dental compensation, the most appropriate operative management is Le Fort I osteotomy with maxillary impaction. Genioplasty may also be performed in patients with persistent retrusion of the chin.

Anterior segmental maxillary osteotomy will not completely correct the long face deformity. Le Fort I osteotomy with inferior repositioning will increase the length of the lower face and thus worsen the deformity. A Le Fort III osteotomy, which entails detachment of the entire midface from the base of the skull, is inappropriate for the treatment of vertical maxillary excess. Sagittal split osteotomy is performed for correction of mandibular deficiencies, not maxillary excess.

97
Q

A 19-year-old man has midface hypoplasia, maxillary retrusion, and the appearance of mandibular prognathism. On examination, he has Angle class III malocclusion and a negative overjet of 15 mm. Which of the following is the most appropriate management?

(A) Anterior mandibular subapical osteotomy
(B) Genioplasty
(C) Le Fort I osteotomy
(D) Sagittal split osteotomy
(E) Combined Le Fort I osteotomy and sagittal split osteotomy

A

The correct response is Option E.

In this patient who has maxillary hypoplasia and mandibular hyperplasia, resulting in midface retrusion and the appearance of mandibular prognathism, the most appropriate management is combined Le Fort I and sagittal split osteotomies. There are several indications for combined surgery in this patient, including correction of the 15 mm of negative overjet and rotation of the maxillary midline. Maxillomandibular correction can be beneficial in patients who have more than 10 mm of either positive or negative overjet.

Anterior mandibular subapical osteotomy can be used to level the occlusal plane, correct asymmetry, or change the anterior positioning and/or axial angulation of the anterior mandibular teeth. Genioplasty alone will not correct this patient’s Angle class III malocclusion. If performed as a single procedure, neither Le Fort I osteotomy nor sagittal split osteotomy will correct both of this patient’s deformities.

98
Q

A 26 year old woman comes to the office because she has had difficulty opening her mouth fully for the past three months. She says she has had €œpopping and clicking € in the temporomandibular joints (TMJs) with pain of varying severity for more than five years. Which of the following is the most likely cause of the internal derangement of the TMJs?

(A) Anterior dislocation with perforation

(B) Anterior dislocation with reduction

(C) Anterior dislocation without perforation

(D) Anterior dislocation without reduction

A

The correct response is Option D.

The patient has an anterior dislocation without reduction. €œPopping and clicking € in the TMJs are due to an anterior relationship of the articular disc to the mandibular condyle. A reducible disc is associated with essentially normal range of motion as translation is allowed to occur. A nonreducible disc maintains an anterior position and does not allow translation to occur fully. Thus, hypomobility of the mandible results.

99
Q

Which of the following orthognathic movements is the most unstable and prone to relapse?
A) Mandibular advancement
B) Mandibular narrowing
C) Maxillary advancement
D) Maxillary widening
E) Sliding genioplasty

A

Correct answer is D.

Transverse widening of the maxilla is the most unstable orthognathic movement. With this procedure, a patient may lose as much as 50% of the movement at one year after surgery. Maxillary downgrafts and mandibular setbacks are also relatively unstable procedures. Mandibular advancement, mandibular narrowing, maxillary advancement, and sliding genioplasty are all considered stable movements.

100
Q

In a patient undergoing orthognathic surgery with preoperative class III malocclusion, which of the following anatomic relationships must be retained in order to optimize postoperative occlusion?

A) Centric relation and centric occlusion
B) Condylar seating
C) Maximal intercuspation
D) 2 mm of overbite
E) 2 mm of overbite and 2 mm of overjet

A

The correct response is Option A.

Centric occlusion is incorrect because centric occlusion (maximal intercuspation) without centric relation (condylar seating within the glenoid fossa) will not lead to reliable postoperative occlusion.

Maxillary occlusion plane angle is incorrect because the maxillary plane angle affects open bite tendency and is not a reliable measure of occlusion.

Gonial angle is incorrect because the gonial angle along with the mandible occlusal plane are better predictors of prognathism and open bite tendency and are not a reliable measure of occlusion.

Centric relation is incorrect because centric relation without centric occlusion will not lead to reliable postoperative occlusion.

Centric occlusion and centric relation is the correct response because maximal intercuspation (centric occlusion) coupled with proper mandible condylar position within the glenoid fossa (centric relation) is most likely to result in optimal occlusion after orthognathic surgery.

101
Q

A 30-year-old woman comes to the office to discuss surgical augmentation of the chin. Which of the following outcomes is most likely in this patient if a porous polyethylene prosthesis is used instead of a solid silicone rubber prosthesis?

A) Increased incidence of bone resorption
B) Increased incidence of infection
C) Increased ingrowth of tissue
D) Increased likelihood of malposition
E) Reduced resorption of the implant

A

The correct response is Option C.

Porous polyethylene implants have enough rigidity to resist soft-tissue deforming forces but enough flexibility to facilitate placement. The pore size (diameter of 100 to 250 ?m) of porous polyethylene used in facial augmentation procedures is sufficient to allow fibrous tissue ingrowth and relative incorporation of the prostheses. This avoids the capsule formation intrinsic to smooth-surface implants which is the result of the host’s foreign body response. This superficial tissue integration makes porous polyethylene facial prostheses less likely to migrate after implantation than solid silicone prostheses, but it also makes their explantation more difficult compared with solid silicone prostheses.

Silicone rubber has a smooth surface and is relatively flexible, making implant placement and removal beneath the soft-tissue envelope easier.

Neither porous polyethylene nor silicone prostheses are resorbed after implantation. Two recent studies reported on a total of 53 patients undergoing chin augmentation with silicone implants. The authors found 55% of those patients experienced underlying bone resorption during the 20 month follow-up period based on lateral radiographs.

102
Q

A 27-year-old man has increased lower incisal show on physical examination. Three months ago he underwent horizontal sliding genioplasty with anterior displacement (8 mm) of the chin followed by fixation with three 24-gauge steel wires.

Which of the following is the most likely cause of his current findings?

(A) Excessive traction on the lower lip
(B) Inadequate fixation of the osteotomy segment
(C) Inadequate repair of the mentalis muscle
(D) Injury to the anterior belly of the digastric muscle
(E) Injury to the mental nerve

A

The correct response is Option C.

The most likely cause of the increased lower incisor show in this patient who underwent horizontal sliding genioplasty, as well as in patients who undergo placement of a chin implant, is improper or inadequate repair of the mentalis muscle. Whenever a lower buccal sulcus incision is made, the mentalis muscle is the first structure seen in the midline after the mucosa is divided. It must be divided properly in order to obtain exposure of the symphyseal mandible for either osteotomy or implant placement. It is essential to precisely close the muscle as a separate layer in order to prevent downward displacement of the lower lip as healing occurs.

Excessive traction on the lower lip usually has no long-term sequelae. Inadequate fixation of the osteotomy segment is an unlikely cause of this patient’s increased incisal show because the three 24-gauge steel wires would have provided adequate fixation for horizontal genioplasty (plate and screw fixation could also have been used). Injury to the anterior belly of the digastric muscle would be more likely to result in necrosis of the advanced chin segment because this muscle supplies blood to the chin segment after horizontal osteotomy. Injury to the mental nerve would cause numbness of the lower lip on the side of the injury.

103
Q

A 13-year-old boy who underwent repair of bilateral cleft lip at 3 months of age and repair of cleft palate at 9 months of age is being evaluated after alveolar bone grafting. He has undergone orthodontic treatment, but a 12-mm negative overjet remains. A photograph is shown. Which of the following operative procedures is most appropriate?
(A) Le Fort I osteotomy with distraction
(B) Le Fort I osteotomy with immediate advancement
(C) Le Fort III osteotomy with distraction
(D) Le Fort II osteotomy with immediate advancement

A

The correct response is Option A.

This patient has severe maxillary retrusion associated with bilateral cleft lip and palate. He has undergone bone grafting and orthodontic treatment, and his deformity is at the Le Fort I level, involving the tooth-bearing maxilla. Distraction osteogenesis at the Le Fort I level has become the mainstay for managing severe maxillary retrusion associated with cleft lip and palate. Before the advent of distraction, traditional Le Fort I advancement would give inadequate advancement due to palatal scarring, and many surgeons simultaneously performed mandibular setbacks to obtain class I occlusion at the expense of facial aesthetics. With distraction, significant advancements can be accomplished. If the osteotomy is performed at the Le Fort III level, vertical elongation of the orbit and resultant enophthalmos occur.

104
Q

Which of the following best describes the normative percentage of the lower third of the face from subnasal to menton when compared to the upper face and mid face?

A) 10%
B) 20%
C) 30%
D) 40%
E) 50%

A

Please note: Upon further review, this item was not scored as part of the examination.

The correct response is Option C.

The face may be divided into horizontal thirds. The upper third extends from the hairline to the glabella, the middle third from the glabella to the subnasale and the lower third from the subnasale to the menton. These facial thirds are rarely equal. If proportions are to be used in orthodontic/orthognathic surgical planning, they should be used only as general guidelines alongside other well-established treatment planning methods.

105
Q

In distraction osteogenesis of the mandible, which of the following is a disadvantage of an external device compared with an internal device?

(A) Ability to alter the vector of distraction

(B) Ability to manipulate the bony regenerate

(C) Greater expansion length

(D) Shorter moment arm at the device €“bone interface

A

The correct response is Option D.

Distraction osteogenesis of the mandible may be performed using external devices or internal devices. External devices are secured to the bony segments via percutaneous pins. Internal devices are placed subperiosteally, typically via an intraoral incision, and are anchored to the bone with screws passing through foot plates.

Advantages of external devices include the potential for three €‘dimensional distraction, the ability to €œmold € the bony regenerate, adjust dental relationships prior to consolidation, a greater length of distraction, and greater ease of placement.

Disadvantages include creation of more prominent facial scars and an increased distance from the body of the distractor to the bone surface. The latter leads to a longer €œmoment arm € at the device €“bone interface and an increased possibility of pin loosening. Internal devices avoid the creation of a significant facial scar, and are much more inconspicuous and cumbersome. They do not allow for alteration of the distraction vector after the device is placed and generally the length of distraction is less.

106
Q

Which of the following is the most important clinical measurement when planning vertical maxillary changes?

A ) Gingival exposure with smiling

B ) Upper incisor exposure with the lips in repose

C ) Upper incisor exposure with smiling

D ) Vermilion exposure with the lips in repose

A

The correct response is Option B.

Upper incisor exposure with the lips relaxed is the key measurement when planning surgical vertical changes of the maxilla, aiming for a range of 3 to 5 mm. Males typically show less than females. Aging also results in a progressive decrease in upper incisor display. Normally, 100% of maxillary incisor is displayed on smile, but this relationship can be quite variable. Using relationships during smiling to determine surgical movements may result in unfavorable lip-tooth aesthetics at rest.

107
Q

A 17-year-old boy is referred for consultation about correction of a bilateral crossbite deformity. The patient has an otherwise normal facial profile. A lateral cephalogram taken with the mouth in repose indicates 2 mm of maxillary incisor show. Which of the following best describes this finding?

(A) Horizontal maxillary deficiency

(B) Horizontal maxillary excess

(C) Normal incisor display

(D) Vertical maxillary deficiency

(E) Vertical maxillary excess

A

The correct response is Option C.

Incisor display of 1 to 4 mm is considered normal.

Horizontal maxillary discrepancies generally are noted in conjunction with posterior crossbites. The horizontal maxillary deficiency is the most common transverse maxillary discrepancy. Horizontal maxillary excess is unusual and would be cause for evaluating the mandible for transverse deficiency.

Lack of incisor display in repose generally indicates vertical maxillary deficiency. Incisor display of greater than 4 mm, especially in male patients, is considered to be associated with vertical maxillary excess. Up to 5 mm of incisal show is considered aesthetically acceptable in female patients. When smiling, the entire clinical crown is normally displayed from the gingival enamel junction to the incisal edge. More or less display with smiling may also be associated with vertical maxillary excess or deficiency.

108
Q

In a 20-year old man, examination of the occlusion shows that the mesial buccal cusp of the maxillary first molar articulates between the first and second mandibular molars. Cephalometric analysis shows SNA angle of 70 degrees (normal 80 to 82 degrees) and SNB angle of 79 degrees (normal 79 to 80 degrees). Which of the following is the most likely diagnosis?
A) Long face syndrome
B) Maxillary deficiency
C) Maxillary excess
D) Retrognathia
E) True prognathism

A

Correct answer is B.

In a Class III mesioclusion, the mesial buccal cusp of the maxillary first molar articulates between the first and second mandibular molars. The mandible is mesially placed or prognathic compared to the maxilla. This type of malocclusion may result from maxillary deficiency, mandibular overgrowth, or a combination of both.
Long face syndrome results from vertical maxillary excess. Physical findings include excessive gingival and upper incisor show at rest and during smiling, a long vertical face, and a retrognathic mandible caused by backward autorotation or true retrognathism.

At rest, normal incisor show is approximately 2 to 3 mm. Decreased incisor show usually indicates maxillary hypoplasia. Excessive gingival show indicates excess maxillary growth.

This patient has a normal SNB angle, indicating that he does not have true prognathism but that it only appears excessive because the maxilla is retrusive. The SNA angle (normally 82 degrees) relates the maxilla to the cranial base. The SNB angle (normally 79 degrees) relates the mandible to the cranial base.

109
Q

A 50-year-old man comes to the office because of a clicking sensation when he opens his mouth. He sustained injuries to the right side of his face 1 year ago when he was involved in a snowmobile collision. Evaluation at that time showed no evidence of a fracture. Current physical examination shows adequate range of motion of the mandible. Which of the following is the most likely cause of the condition described?

A) Blood within the temporomandibular joint capsule
B) Contact of the condylar head against the articular eminence
C) Contact of the mandibular condyle against the glenoid fossa
D) Foreign body in the temporomandibular joint
E) Subluxation of the articular disk

A

The correct response is Option E.

A “clicking sensation” upon opening the jaw is commonly caused by anterior subluxation of the articular disk. The temporomandibular joint is unique in that it is a diarthrodial joint, having one space above the disk and a second space below. Clicking occurs when the posterior attachments of the disk become attenuated or ruptured. The disk is then allowed to sublux anteriorly and then relocate. Patients with painless clicks may be observed; those with painful clicking during jaw closure require treatment. Conservative management consists of NSAID therapy, the use of a bite block, and aggressive physical therapy. More aggressive management includes surgery, which is reserved for those who fail conservative therapy. The other causes are much less likely.

110
Q

A 23-year-old woman with vertical maxillary excess is scheduled to undergo a LeFort I osteotomy with impaction. During this procedure, which of the following pairs of arteries is most likely to be disrupted?

A) Ascending pharyngeal and ascending palatine
B) Greater palatine and facial
C) Maxillary and lesser palatine
D) Nasopalatine and descending palatine
E) Sphenopalatine and pterygopalatine

A

The correct response is Option D.

Understanding the vascular anatomy of the maxilla is critical to successfully performing a Le Fort I osteotomy. In addition to the periosteal blood supply from the mucosal attachments, there are several named vessels that supply the palate and alveolar processes, which are mobilized in a Le Fort I osteotomy. Of the answer choices listed, the nasomaxillary and descending palatine arteries are commonly disrupted during surgery. After downfracture, the ascending palatine and ascending pharyngeal arteries become the dominant blood supply to the segment.

111
Q

A 24-year-old woman with maxillary hypoplasia is scheduled to undergo Le Fort I osteotomy. In order to protect the maxillary dentition, the osteotomy must be carried out above the dental apices. Which of the following maxillary teeth have the longest roots?

(A) Central incisors
(B) Lateral incisors
(C) Cuspids
(D) First bicuspids
(E) First molars

A

The correct response is Option C.

The cuspids, or canine teeth, have the longest roots in both the maxilla and mandible. The average length of a cuspid tooth from the tip of the root to the tip of the crown is 27 mm.

Knowledge of the length and position of the dental roots will help to prevent injury during Le Fort I osteotomy and placement of internal fixation during fracture reduction. The dentition can also be injured during stabilization of maxillary or mandibular fractures.
The average length of a molar tooth from the tip of the root to the tip of the crown is 24 mm. The length of the upper first molar is 21.3 mm.

The cuspid tooth is named for its single cusp; bicuspids have two cusps (buccal and lingual), and molars have three cusps (mesiobuccal, distobuccal, and mesiolingual). The average adult has two maxillary cuspids, four maxillary bicuspids, and six maxillary molars.

112
Q

A 28-year-old woman who underwent Le Fort I osteotomy six weeks ago comes to the office for follow-up evaluation. She says her nose is now wider than it was before the procedure. Addition of which of the following interventions to the osteotomy procedure would have effectively minimized this adverse result?

(A) External splinting of the nose
(B) Placement of an alar cinch suture
(C) Modified Weir excision
(D) Reduction of the amount of piriform rim exposure
(E) V-Y advancement closure of the lip

A

The correct response is Option B.

Placement of an alar cinch suture helps decrease the degree of widening of the alar base that occurs after exposure of the anterior maxilla for orthognathic surgery or trauma management. This suture is placed in the base of the ala bilaterally and then is tightened until the desired effect is achieved.

External splinting of the nose has no effect on dimensional changes of the nasal ala. Although a modified Weir excision is designed to treat alar flare, it does not address the increased width of the alar base seen after Le Fort I osteotomy. Reduction of piriform rim exposure is not appropriate for this patient. During Le Fort I osteotomy, complete exposure of the piriform rim is essential because the rim serves as a landmark from which to measure movement in the maxilla. Also, the bone in the piriform rim and the malar buttress provides a stable platform for rigid fixation.

V-Y advancement is used during oral mucosal closure, especially after Le Fort I osteotomy, which tends to flatten the upper lip. This suture technique advances the tissue anteriorly to add fullness to the upper lip, but has no effect on nasal width.

113
Q

A 24 year old woman comes to the office because she has had popping of the left temporomandibular joint and associated mild pain that have been worsening over the past six months. The patient says she also has intermittent inability to fully open her mouth, but she does not believe her symptoms restrict her daily routine. Physical examination shows mild bilateral tenderness of the preauricular region but no popping, clicking, or crepitus. Maximal incisal opening is 20 to 25 mm, and lateral and protrusive excursions are reduced. MRI of the temporomandibular joints shows mild anterior dislocation of the left meniscus with reduction in the open mouth position. Which of the following is the most appropriate initial management?

(A) Administration of a nonsteroidal anti-inflammatory drug, occlusal splint therapy, and restriction to soft diet

(B) Comprehensive orthodontic therapy with bilateral sagittal ramus osteotomies of the mandible

(C) Initial intra-articular injection of a corticosteroid followed by serial injections if symptoms do not resolve

(D) Open arthrotomy with meniscectomy followed by a brief period of joint immobilization and then physical therapy

(E) Open arthrotomy with plication of the meniscus followed by a brief period of joint immobilization and then physical therapy

A

The correct response is Option A.

With symptoms and clinical findings of mild pain and popping of the temporomandibular joint, mild tenderness of the preauricular region, and transient joint locking, the patient described has an early/intermediate stage internal derangement. MRI shows only mild changes in meniscal position. Therefore, conservative therapy such as a soft diet, use of nonsteroidal anti-inflammatory drugs, and splinting is indicated. Most patients with temporomandibular joint disorders will achieve some relief of symptoms with nonsurgical therapy within a period of two to three years. However, the pathologic process may continue, and follow €‘up is recommended.

114
Q

An 18-year-old woman who underwent repair of unilateral cleft lip and palate in infancy comes to the office for consultation regarding facial aesthetics. Cephalometric analysis shows an SNA angle of 75 degrees (N 81.2), an SNB angle of 81 degrees (N 77.3), and an SNPg angle of 81 degrees (N 80). A negative overjet of 1.5 mm is noted. A photograph is shown. After orthodontic preparation, which of the following is the most appropriate surgical management?

A) Maxillary advancement
B) Maxillary advancement and mandibular setback
C) Maxillary advancement, mandibular setback, and advancement genioplasty
D) Maxillary setback
E) Maxillary setback, mandibular advancement, and advancement genioplasty

A

The correct response is Option C.

The cephalogram is a standardized x-ray study used for analyzing facial disharmony and asymmetry. Labeled landmarks help establish the relationship of the maxilla, mandible, and skull base to other facial structures. The sella (S) point marks the center of the hypophyseal fossa. The nasion (N) is the junction of the nasal and frontal bones at the most posterior point of the curve of the nose. Point ?A? marks the innermost curvature from the maxillary anterior nasal spine to the alveolar process. The angle created by these points (SNA) establishes the maxillary position in relation to the skull base. Point ?B? is located at the innermost curvature from the chin to the alveolar process of the mandible. The SNA angle similarly establishes the mandibular relationship to the skull base.

The prominence of the chin is often an important consideration in orthognathic surgery. The pogonion is the most anterior chin point. SNPg angle is representative of the degree of chin prominence relative to the cranial base (mandible position).

Normal angle values are given in the text. The patient described is in a dramatic Angle class III malocclusion. The SNA is less than normal, and the SNB is greater than normal. The chin position is appropriate. In order to establish a class I dental relationship, the maxilla must advance, and the mandible must be set back. Doing so will also move the chin back. Therefore, to maintain the preoperative chin position, a compensatory advancement genioplasty should also be performed.

115
Q

A 38-year-old woman is referred to the office by an orthodontist for orthognathic surgery to correct facial disharmony. The following angles are noted on cephalometric analysis:

SNA 80 degrees (N 81.2)

SNB 76 degrees (N 77.3)

SN-Pogonion 70 degrees (N 80)

Which of the following is the most appropriate surgical procedure?

A ) Advancement genioplasty

B ) Le Fort I advancement and advancement genioplasty

C ) Le Fort I advancement and setback genioplasty

D ) Sagittal split mandibular advancement

E ) Sagittal split mandibular setback

A

The correct response is Option A.

The cephalogram is a standardized radiograph used for analyzing facial disharmony and asymmetry. Labeled landmarks help establish the relationship of the maxilla, mandible, and skull base to other facial structures. The sella (S) point marks the center of the hypophyseal fossa. The nasion (N) is the junction of the nasal and frontal bones at the most posterior point of the curve of the nose. The A point marks the innermost curvature from the maxillary anterior nasal spine to the alveolar process. The angle created by these points (sella-nasion angle, SNA) establishes the maxillary position in relation to the skull base. The B point is located at the innermost curvature from the chin to the alveolar process of the mandible. The SNA angle similarly establishes the mandibular relationship to the skull base.

The prominence of the chin is often an important consideration in orthognathic surgery. The pogonion (Pg) is the most anterior chin point. The sella-nasion-pogonion (SNPg) angle is representative of the degree of chin prominence relative to the SNB (mandible position).

Normal angle values are given in the scenario. The patient described has a relatively retrusive pogonion and a normally positioned mandible/maxilla. This would represent a class I dental relation. The pogonion is posteriorly displaced in relation to both the mandible and the maxilla.

Therefore, to establish better facial relationships, the chin prominence should be advanced via a sliding genioplasty. The mandible does not need to move.

116
Q

In patients with vertical maxillary excess undergoing Le Fort osteotomy with maxillary impaction, which of the following findings is most likely postoperatively?
(A) Increased mentalis strain
(B) Increased upper incisal show
(C) More obtuse nasolabial angle
(D) Retrogenia
(E) Widened alar base

A

The correct response is Option E.

Patients with vertical maxillary excess, or long face syndrome, have a narrow alar base, an obtuse nasolabial angle, and an anterior open bite. Mentalis muscle strain and labial incompetence are increased, and there is excess gingival show and exposure of the upper incisors.

Appropriate management is Le Fort I osteotomy with maxillary impaction; osseous genioplasty is also performed in some patients. These procedures will correct many of the findings associated with this condition, including decreasing the mentalis muscle strain and incisal show and creating a more acute nasolabial angle. The alar base will be widened. Le Fort I osteotomy also rotates the mandible forward and upward, resolving the retrogenia associated with long face syndrome. Postoperative lateral cephalograms will show forward autorotation of the mandible with counterclockwise rotation.

117
Q

A 24-year-old woman comes to the office for consultation regarding her appearance when smiling. Physical examination shows the mesiobuccal cusp of the first maxillary molar lying distal to the buccal groove of the first mandibular molar. Which of the following is the most appropriate Angle classification?
(A) I
(B) IIA
(C) IIB
(D) III

A

The correct response is Option D.

Occlusion as described by the Angle classification uses the relationship of the permanent first molars of the maxilla and mandible as its reference point. Patients with class I occlusion (normal) have the mesiobuccal cusp of the maxillary first molar lying in the buccal groove of the mandibular first molar. Patients with class II occlusion have the mesiobuccal cusp of the maxillary first molar located mesial to the buccal groove of the mandibular first molar. Class III occlusion, as illustrated in this case, is described as having the mesiobuccal cusp of the maxillary first molar positioned distal to the buccal groove of the mandibular first molar.

118
Q

A 27-year-old man has articulation of the mesiobuccal cusp of the first upper molar with the distobuccal groove of the lower first molar. Cephalometric analysis shows increased SNB angle and negative ANB angle. Which of the following interventions are the most appropriate management of this patient’s facial deformity?

(A) Fronto-orbital advancement and Le Fort I advancement
(B) Le Fort I setback and jumping genioplasty
(C) Le Fort III advancement and mandibular advancement
(D) Maxillary impaction and vertical reduction advancement genioplasty
(E) Sagittal split osteotomy and maxillary advancement

A

The correct response is Option E.

This patient has mandibular prognathism, which is treated with sagittal split osteotomy (mandibular setback) and maxillary advancement. Physical examination shows Angle class III malocclusion, and cephalometric analysis shows excessive protrusion of the mandible in relation to the maxilla and base of the cranium. Although mandibular setback alone may seem to be the most logical treatment, most patients with mandibular prognathism require treatment with a combination of mandibular setback and maxillary advancement. This corrects the projecting mandible and fills the soft-tissue envelope, creating a better aesthetic result.

In a patient with mandibular prognathism, fronto-orbital advancement and Le Fort I advancement would not treat the mandibular protrusion. Le Fort I setback and jumping genioplasty also would not correct the mandibular protrusion because they do not address the mandible as a whole. Le Fort III advancement and mandibular advancement would worsen the prognathic mandible. Maxillary impaction and vertical reduction advancement genioplasty do not treat the malocclusion or mandibular skeletal abnormality.

119
Q

Which of the following best describes the occlusal relationship indicated by the arrow labeled B in the diagram shown?

A ) Crossbite

B ) Open bite

C ) Overbite

D ) Overjet

A

The correct response is Option D.

Overjet is the horizontal relationship between the maxillary and mandibular incisors shown in the diagram. Crossbite describes a reverse relationship between the maxillary and mandibular teeth, either

sagittally or buccolingually. Open bite refers to the lack of vertical overlap by the maxillary teeth over the mandibular teeth. Overbite is the vertical overlap between the maxillary and mandibular incisors.

120
Q

During a Le Fort I osteotomy, the descending palatine artery is disrupted and ligated. The blood supply to the mobilized maxilla is primarily from which of the following arteries?

A) Ascending pharyngeal
B) Greater palatine
C) Internal maxillary
D) Lesser palatine

A

The correct response is Option A.

The vascular supply of the Le Fort I osteotomy segment was studied by utilizing standard latex injection techniques. Anatomic dissections of 10 fresh cadavers demonstrated interruption of the descending palatine arteries with preservation of the ascending palatine branch of the facial artery and the anterior branch of the ascending pharyngeal artery within the attached posterior palatal soft-tissue pedicle in all specimens following Le Fort I maxillary osteotomy. These ascending arterial branches entered the soft palate at a position approximately 1 cm posterior to the pterygomaxillary junction, which was disrupted during the Le Fort I maxillary osteotomy. Separate ink injections of total maxillary osteotomy segments confirmed vascular perfusion of the ipsilateral hemimaxillary segment by the ascending palatine artery. Thus vascular supply of the mobilized Le Fort I maxillary segment is by means of the ascending palatine branch of the facial artery and the anterior branch of the ascending pharyngeal artery in addition to the rich mucosal alveolar anastomotic network overlying the maxilla.

121
Q

Which of the following terms best describes the temporomandibular joint?

A) Ellipsoid (condyloid)
B) Gliding (arthrodial)
C) Hinge-sliding (ginglymoarthrodial)
D) Pivot (trochoid)
E) Saddle (ephippial)

A

The correct response is Option C.

The temporomandibular joint is classified as a ginglymoarthrodial joint since it has both hinge and sliding components during jaw opening. These functions take place in the two separate compartments in the joint, upper and lower, that are effectively separated by an articular disc. During the first 20 mm of jaw opening, the condyle rotates in the lower compartment (space between condylar head and articular disc) in a nearly pure hinge motion. For further opening to take place, the condyle translates (or shifts) forward with the articular disc through the upper compartment (space between the articular disc and the joint surface). The other options describe other joint configurations. Examples of each are: saddle, thumb basilar joint; pivot, atlas-axis (C1-2 neck); gliding, tarsal bones in the foot; ellipsoid, radiocarpal articulation.

122
Q

During genioglossus advancement, which of the following nerves innervates the affected muscle?

A) Facial nerve
B) Glossopharyngeal nerve
C) Hypoglossal nerve
D) Lingual nerve
E) Vagus nerve

A

The correct response is Option C.

During genioplasty or horizontal osteotomy of the mandible, the genioglossus muscle is identified after down fracture and separation of the mobile and nonmobile segments. The genioglossus muscle is not only a source of blood supply but also acts to advance the tongue during skeletal sleep apnea surgery and contributes to posterior pull on potential relapse of the mobilized chin segment. This important muscle is innervated by the hypoglossal nerve.

123
Q

Which of the following cephalometric landmarks is included in the Frankfort horizontal plane?

A) Nasion
B) Pogonion
C) Point B
D) Porion
E) Sella turcica

A

The correct response is Option D.

The two cephalometric planes used most frequently in lateral cephalograms to describe and evaluate the cranial base are the Sella-nasion plane and the Frankfort horizontal plane. The Frankfort horizontal plane is defined by a line from the superior edge of the external auditory meatus (porion) to the inferior orbital meatus (orbitale). The SNA and SNB are angles used to describe the position of the maxilla and mandible, respectively. The SN refers to a line from the sella turcica to the nasion, while point A is on the maxilla and point B is on the mandible. The pogonion refers to the chin point.

124
Q

A 12-year-old girl is evaluated because of a 1-year history of progressive mandibular retrognathia and bilateral temporomandibular joint pain. There is no history of trauma. Examination shows slightly decreased interincisal opening without chin point deviation and an Angle class II malocclusion with an anterior open bite. Which of the following is the most likely cause of this patient?s symptoms?

A ) Bruxism
B ) Condylar hyperplasia
C ) Infection
D ) Myofascial pain syndrome
E ) Rheumatoid arthritis

A

The correct response is Option E.

Disorders that can affect the temporomandibular joint (TMJ) include ankylosis, arthritis, trauma, dislocation, congenital and developmental anomalies, and neoplasms. Rheumatoid arthritis (RA) can cause tenderness, swelling, and decreased motion in any joint, including the TMJ. The TMJ can be affected in up to 33% of patients with RA. Chronic inflammation can, eventually, result in articular erosions, joint destruction, and ankylosis. When RA develops in childhood or early adolescence (juvenile idiopathetic arthritis), erosion of the condyles can lead to progressive mandibular retrognathism and anterior open bite.

Bruxism is grinding of the teeth and can lead to progressive dental wear, myofascial pain, and TMJ derangement. It does not affect the facial profile. Condylar hyperplasia is overgrowth of the condyle. It is most commonly unilateral, painless, and can lead to chin point deviation. Infection can cause tenderness and, ultimately, degeneration of the TMJ. However, infection is rarely bilateral and there are usually concurrent systemic symptoms. Myofascial pain syndrome is a common cause of pain in the TMJ region. It is more common in girls and is considered a localized form of fibromyalgia in the head and neck. It usually is unilateral and does not typically lead to joint degeneration or alterations in occlusion.

125
Q

A 13-year-old boy who underwent repair of bilateral cleft lip at 3 months of age and repair of cleft palate at 9 months of age is being evaluated after alveolar bone grafting. He has undergone orthodontic treatment, but a 12-mm negative overjet remains. A photograph is shown. Which of the following operative procedures is most appropriate?
(A) Le Fort I osteotomy with distraction
(B) Le Fort I osteotomy with immediate advancement
(C) Le Fort III osteotomy with distraction
(D) Le Fort II osteotomy with immediate advancement

A

The correct response is Option A.

This patient has severe maxillary retrusion associated with bilateral cleft lip and palate. He has undergone bone grafting and orthodontic treatment, and his deformity is at the Le Fort I level, involving the tooth-bearing maxilla. Distraction osteogenesis at the Le Fort I level has become the mainstay for managing severe maxillary retrusion associated with cleft lip and palate. Before the advent of distraction, traditional Le Fort I advancement would give inadequate advancement due to palatal scarring, and many surgeons simultaneously performed mandibular setbacks to obtain class I occlusion at the expense of facial aesthetics. With distraction, significant advancements can be accomplished. If the osteotomy is performed at the Le Fort III level, vertical elongation of the orbit and resultant enophthalmos occur.

126
Q

A 30-year-old woman has painful clicking of the jaw six months after sustaining blunt trauma to the face in a motor vehicle collision. Radiographs taken at the time of injury showed no evidence of fracture. Which of the following is the most appropriate diagnostic study in the evaluation of this patient?

(A) Arthroscopy
(B) CT scan
(C) Digital subtraction angiography
(D) MRI
(E) Tomography

A

The correct response is Option D.

MRI is the most appropriate diagnostic study in the evaluation of this patient who has painless clicking of the jaw after sustaining blunt trauma to the face. In fact, MRI is preferred for evaluation of all conditions involving the articular disk and temporomandibular joint (TMJ), including internal derangement of the TMJ. This painless noninvasive modality allows examination in multiple planes and produces an accurate visualization of the soft tissues without exposing the patient to ionizing radiation. Fast-scanning techniques can be used to assess TMJ pathology.

Arthroscopy involves insertion of a scope into the joint space to allow for direct visualization of internal structures. Although this more invasive procedure allows surgeons to visualize the TMJ, it puts the patient at increased risk for the development of adhesions.

CT scans are more effective than conventional radiographs for the evaluation of fractures and tumors within the jaws; they can also be used to evaluate the bony structures of the TMJ. However, this modality exposes the patient to ionizing radiation and provides a less accurate representation of the soft tissues.

Digital subtraction angiography is used for visualization of the vascularity of the head and neck and not the soft tissues of the TMJ.

Tomography has been replaced by the more accurate CT scan and MRI, and is no longer used.

127
Q

Which of the following structures is NOT involved in a Le Fort II advancement osteotomy?

(A) Lateral orbital wall
(B) Medial orbital walls
(C) Nasofrontal junction
(D) Orbital floor
(E) Pterygoid plates

A

The correct response is Option A.

The Le Fort classification is used to identify the pattern of midface fractures. Identification of the lines of fracture is useful in planning osteotomies for patients requiring midface advancement.

Le Fort I midface advancement involves placement of the osteotomy at a level above the apices of the teeth. The entire alveolar processes of the maxilla, vault of the palate, and pterygoid processes are included in a single block. The osteotomy extends transversely across the base of the maxillary sinuses and the floor of the piriform aperture.
The Le Fort II osteotomy begins above the level of the apices of the teeth laterally and extends through the pterygoid plates in a manner similar to the Le Fort I osteotomy, leaving a central maxillary segment undisturbed. The osteotomy procedure includes portions of the medial orbital wall, orbital floor, and nasofrontal junction. Patients undergoing Le Fort II midface advancement are at increased risk for injury to the ethmoid area and the lacrimal system. The lateral orbital wall is unaffected.

Le Fort III osteotomy extends through the zygomaticofrontal suture and the nasofrontal suture and across the floor of the orbits. The entire midface is completely detached from the base of the skull.

128
Q

Interorbital distance is most accurately determined by measuring the distance between which of the following structures?

(A) Anterior lacrimal crests

(B) Inferior orbital fissures on a plain anteroposterior cephalogram

(C) Lateral aspects of the medial canthi on standard anteroposterior photograph

(D) Pupils on a standard anteroposterior photograph

(E) Supraorbitale (SOr) €” the most anterior point of the intersection of the orbital roof and its lateral contour

A

The correct response is Option A.

By convention, the interorbital distance is determined as the shortest distance between the medial walls of the orbit. This usually falls at the dacryon, which refers to the craniometric point at the junction of the anterior border of the lacrimal bone with the frontal bone. The normal distance is approximately 25 mm in women and 28 mm in men.

129
Q

Three weeks after undergoing bilateral maxillary advancement, a 28-year-old man has exposure of two of the four maxillary plates in the oral cavity. The maxilla appears stable. Which of the following is the most appropriate next step in management?

(A) Instruction in oral hygiene and observation
(B) Irrigation and debridement of the wound followed by replacement of the two exposed plates
(C) Maxillomandibular fixation with heavy elastic for two weeks
(D) Removal of all plates followed by intermaxillary fixation for six weeks
(E) Removal of the two exposed plates only

A

The correct response is Option A.

In this patient who has exposure of the internal hardware, the most appropriate management is maintenance of optimum oral hygiene. As long as oral hygiene is maintained, the oral mucosa is likely to granulate over the maxillary plates. After satisfactory bone healing has been achieved, any exposed plates can be removed.

Removal of the hardware is not indicated in a patient who underwent surgery only three weeks earlier because there is an increased risk of bony malunion. The maxilla is stable, and maxillomandibular fixation is unnecessary. The patient can continue nutrition with a soft diet without adverse sequelae.

130
Q

Which of the following is the ideal amount of tooth show with the lips in repose?

A ) 2 mm of lower incisor show

B ) 4 mm of lower incisor show

C ) 2 mm of upper incisor show

D ) 4 mm of upper incisor show

E ) No tooth show

A

The correct response is Option C.

The ideal amount of tooth show with the lips in repose is 2 mm of the maxillary incisors. In women, up to 4 mm may be acceptable. However, more show of the upper dentition results in a gummy appearance. One possible etiology for a gummy appearance is vertical excess of the maxilla. Orthognathic surgery with maxillary impaction is the typical treatment for this deformity.

Over time, elongation of the upper lip decreases the amount of show of the upper dentition. Gravitational pull on the lower lip may result in the increased display of the mandibular dentition with advancing age. Techniques such as lip augmentation and lip lift are designed to correct these changes of aging.

131
Q

A 6-year-old boy with hemifacial microsomia undergoes mandibular distraction. Which of the following is the correct sequence of phases in distraction osteogenesis of the mandible?

(A) Activation, consolidation, latency

(B) Activation, latency, consolidation

(C) Consolidation, latency, activation

(D) Latency, activation, consolidation

(E) Latency, consolidation, activation

A

The correct response is Option D.

The correct sequence of phases in distraction osteogenesis of the mandible is latency, activation, and consolidation. Latency is the period immediately following the corticotomy and application of the distraction device. It ranges from one to seven days, during which no movement of the bony segments occurs. After the latency phase is the activation phase, during which the distraction device is turned to lengthen the gap between the bones by 1 mm/d. This is typically done with one 0.5-mm turn twice daily. After the device has been activated to achieve the desired lengthening, the consolidation phase begins. During this time, the device is left in place but not activated. The consolidation phase is generally twice the number of days necessary for the activation phase.

132
Q

A 34-year-old woman desires an improved aesthetic appearance of the chin. On physical examination, she has a bony chin deformity characterized by sagittal deficiency and vertical mandibular excess. She has class I occlusion. Which of the following genioplasty procedures should be performed in this patient?

(A) Asymmetric genioplasty
(B) Interposition genioplasty
(C) Jumping genioplasty
(D) Sliding genioplasty
(E) Silastic implantation

A

The correct response is Option C.

This 34-year-old woman who has a bony chin deformity should undergo jumping genioplasty. In this procedure, the transverse osteotomy is performed initially, decreasing the vertical dimension of the chin; following this, the osteotomized segment is transposed anteriorly with its attached suprahyoid musculature to augment the sagittal deficiency.

Asymmetric genioplasty involves adjusting the lines of osteotomy in multiple vectors in order to correct a misshapen chin. Interposition genioplasty can be performed to increase the vertical and sagittal dimensions of the chin; autogenous bone or hydroxyapatite can be grafted to the osteotomy site. Although sliding genioplasty can be used for correction of a sagittal deficiency or an excessively large chin, the change in vertical chin dimension seen following this procedure is only minimal. Silastic implantation will correct the sagittal deficiency but not the vertical excess.

133
Q

A 4-year-old girl is undergoing mandibular reconstruction involving the temporomandibular joint. Use of which of the following types of bone graft is most likely to result in overgrowth on the reconstructed side?

A ) Calvaria

B ) Fibula

C ) Iliac crest

D ) Radius

E ) Rib

A

The correct response is Option E.

A variety of bone sources may be used in reconstructing the deficient mandible in pediatric patients. The majority of reconstructions are done for congenital anomalies involving the mandible, such as hemifacial microsomia and Treacher Collins syndrome. However, bone grafting may also be needed following tumor resection and traumatic loss.

Cortical bone may be harvested from the iliac crest, calvaria, rib, radius, and fibula. Typically, when rib bone is used to reconstruct the mandible, including the temporomandibular joint, a cartilaginous cap is left on the end of the rib when it is harvested. This allows for growth of the rib as the child grows, but it can also result in overgrowth. This overgrowth can result in further asymmetry and malocclusions.

134
Q

An 18-year-old woman is referred for evaluation of lower facial asymmetry. Examination shows the mandibular dental midline 6 mm to the left of the midsagittal plane, and the chin point and maxilla are coincident with the midsagittal plane. Which of the following procedures is most appropriate to correct the lower facial asymmetry?
A) Le Fort I osteotomy with midline shift to the left and mandibular sagittal osteotomy with shift to the right
B) Mandibular sagittal osteotomy with midline shift to the left
C) Mandibular sagittal osteotomy with midline shift to the right
D) Mandibular sagittal osteotomy with midline shift to the left and sliding genioplasty to the right
E) Mandibular sagittal osteotomy with midline shift to the right and sliding genioplasty to the left

A

Correct answer is E.

In assessing the orthognathic patient preoperatively, it is important to recognize all asymmetries and to be aware of potential asymmetries that may result from planned procedures. Combined osteotomies of the mandible and chin are indicated to bring the mandibular dental midline and the bony chin midline in line with the midsagittal plane in the patient described. To correct the jaw deformity, the mandibular dental midline would have to move to the right. The chin point would also move to the right and would therefore require an osteotomy and left shift.
Combining a Le Fort I and mandibular sagittal osteotomies would result in coincident dental midlines, but neither would be aligned with the midsagittal plane.

Correcting the dental midline alone with mandibular sagittal osteotomies would create a genial asymmetry that was not present preoperatively.

135
Q

An 18-year-old man with a history of cleft lip and palate repair is planning to undergo orthognathic surgery to correct his malocclusion. When counseling the patient regarding complications, which of the following is the most common complication after orthognathic surgery?
A) Dental injury
B) Hardware failure
C) Hemorrhage
D) Infection
E) Nerve injury

A

The correct response is Option E.
Large-scale studies involving hundreds to even thousands of patients have demonstrated the safety and risks associated with orthognathic surgery. Although all of the complications listed in the options have been described during orthognathic surgery, the most common complication is nerve injury ranging from 12.1 to 19%. Infection has been reported from 2 to 3.4%. Hardware failure occurs in up to 2.5% of cases, while dental injuries range from 0.14 to 5%. Significant hemorrhage occurs in 0.5 to 1.4% of cases. Additional complications associated with orthognathic surgery include postoperative temporomandibular joint disorders/condylar disorder, scar problems, and necessity of secondary surge

136
Q

A 23-year-old woman seeks improvement in the appearance of her smile. A Le Fort I osteotomy is planned for correction of maxillary excess. Which of the following arteries is at increased risk for injury during this procedure?

a. Ascending pharyngeal artery
b. Descending palatine artery
c. Infraorbital artery
d. Posterior superior alveolar artery
e. Sphenopalatine artery

A

The correct response is Option B.

The blood supply to the maxilla before Le Fort I osteotomy is from the descending palatine, infraorbital, and posterior superior alveolar arteries, which are branches of the internal maxillary artery off the external carotid artery. When performing a Le Fort I osteotomy, the descending palatine artery runs vertically in the posterior maxilla and is at risk of injury.

After Le Fort I osteotomy, the blood supply to the maxilla is from the ascending palatine artery, which is a branch of the facial artery, off the external carotid artery and the palatine branch of the ascending pharyngeal artery, off the external carotid artery.

Reference(s)

  1. Bykowski MR, Hill A, Garland C, et al. Ruptured pseudoaneurysm of the maxillary artery and its branches following Le Fort I osteotomy: evidence-based guidelines. J Craniofac Surg. 2018 Mar 1.
  2. Bauer RE, Ochs MW. Maxillary Orthognathic Surgery. Oral Maxillofac Surgery Clin North Am. 2014; 26(40):523-37.